You are on page 1of 50

Ray Optics

* Light Lens
" Reflection of Light .. Refraction Through a Prism
Spherical Mirrors ,. Dispersion
" Refraction " Optical Instruments
* Total Internal Reflection Resolving Power of a
.. Refraction from a Spherical Surface " Power of Telescope

t> light
The light is that form of energy which makes objects visible to our
eyes. The branch of physics which deals with nature of light, its sources,
properties, measurement, effects and vision is called "optics". For the sake
of convenience, study of optics is generally divided into two parts namely
(i) geometrical optics or ray optics, and (ii) wave optics. This chapter
deals with the geometrical optics.
*> Reflection of Light
When a beam of light is incident on a polished interface, it is thrown
back in same medium. This phenomenon is called reflection.
In reflection the frequency, speed and wavelength do not change, but
a phase change may occur depending on the nature of reflecting surface.
Experimentally it is found that the rays corresponding to the incident
and reflected waves make equal angles with the normal to the surface.
Thus, the two laws of reflection can be summarized as under.
1. Li = Lr
2. Incitlent ray, reflected ray and normal lie on the same plane.
886 Chapter 23 Ray Optics
J":
>bs0t-,
Normal
.(\iil Note
The above two laws of reflection can b.e appli.ed to the reflecting surfaces
wh}ch a(e not ev.en horizontal.
9rh r(g,.
1t811ection from Plane Mirror (Surface}
In case of reflection from plane surface such as plane mirror
(i) The image is always erect, virtual and of exactly the
same size as the object. The image is formed as much
behind the mirror as the object is in from of it.
(ii) The image is laterally inverted.
(iii) If keeping the incident ray fixed, the plane mirror
is rotilted through an angle e, the reflected ray turns
through double the angle ie, 28 in that very direction.
(iv) If the object is fixed and the mirror moves relative to the
,object with a speed v, the image moves with a speed 2v
to the object.
(v) If the mirror is fixed and the object moves relative to the
mirror with a speed v, the image also moves with the
same speed .. y relative to the mirror.
(vi) Deviation suffered by a light ray incident at an angle i is
given by 8 = (180 - 2i) ,
(vii) If there are two mirrors inclined at an angle 8, the total
number of images formed for an object kept between
the two is equal to
2
8
7t or ( - 1} which ever is odd.
(viii) The minimum size of a mirror required to see the full
image of a person, is half the height of the person.
(ix) If a plane mirror is rotated about an axis perpendicular
to plane of mirror then reflected ray image do not
rotate.
Instance 1 The minimum size of the mirror fixed on the wall of a
room in which an observer at the centre of room may see the full image
of the wall of heightJLbehind him is
(a) (b)
3 2
(c) 2h (d) h
3
lnterp.ret From 0' M
1
M
2
and O'AB
M1M2 =-x-
h 2x +y
h
,
Size of mirror, M M - hx
1 z-(2x+y)
If
Instance 2 Find velocity of image when object and mirror
both are moving toward each other with velocity 2 ms-
1
and 3 ms-
1
respectively .
(a) 8ms-
1
(c) -5ms-
1
Interpret Here
(b) --Bms-1
(d) 5ms-
1
Vo-VM=-(Vl-VM)
=> ( +2ms-
1
)- ( -3ms-
1
) ( -3)
Instance 3 Two plane mirrors are inclined at 30 as shown in
figure. A light ray is incident at angle 45. Find total deviation produced
by combination of mirror after two successive reflection.
(a) 60 i
(c) 50 i
(b) 58 i
(d) 68 i
Interpret Deviation at mirror M
1
,8
1
= 180-2 x 45= 90 i
Deviation at mirror M
2
,8
2
= 180-2 x 15 = 150t
Total deviation 8 = 8
2
-8
1
= 150- 90 = 60 i
Spherical Mirrors
Mirrors having their reflecting surface spherical are called
spherical mirrors. Spherical mirrors are of two types
(i) Concave mirror If reflection takes place from the inner
surface, the mirror is called concave [Fig. (a)].
(ii) Convex mirror If reflection takes place from the outer
surface, the mirror is called convex [Fig. (b)] .
A
Incident
light
---+
Incident
light
-----...--....-=: P----.. +ve
---+ - c
(a) Concave mirror (b) Convex mirror
Definitions of Some Terms Related to Spherical Mirrors
Centre and radius of
curvature The centre of curvature
and radius of curvature of a mirror
are the centre and radius of the
sphere of which the mirror is a part.
In the given figure, AC is the radius
of curvature and C, the centre of
curvature.
Pole Pole of the spherical
mirror is mid point of its reflecting
surface. In figure it is shown by P.
c
Principal ,axis The principal axis of a spherical mirror is
the line joining the pole and centre of curvature. In the figure PC
is principal axis.
Principal focus Principal focus is a point on the principal
axis of the mirror at which the light rays coming parallel
to principal axis actually meet after reflection or appear to
meet.
c
(a) Concave mirror (b) Convex mirror
0
For concave mirror focus is infront of the mirror, while for
convex mirror focus is behind the mirror. Focus of concave mirror
is real, while focus of convex mirror is virtual.
Focal length The distance between pole and focus of
a spherical mirror is called its focal length. It is represented
byf
ie, f = !!_
2
Table 23.1 Image Formation by Concave Mirror
1. At infinity
2.
Between infinity and C
Chapter 23 Ray Optics 887
Ray Tracing
In geometrical optics, to locate the image of an object.
Tracing of a ray as it reflects or refracts, is very important.
1. A ray going through centre of curvature is reflected back
along the same direction.
Concave Convex
~
c
2. A ray parallel to principal axis is reflected through the
focus, and vice-versa. Also, mutually parallel rays. !\{!]1t .
reflection intersect on the focal plane.
F
.E)
3. The light corning through the focus of rriirror or coming
towards focus, becomes parallel to principal axis.
;)
F
Sign Convention for Mirrors
M'
According to the sign convention
(i) Origin should be placed at the pole (P).
(ii) All distances should be measured from the pole (P).
(iii) Object distance is denoted by u, image distance by v,
focal length by f and radius of curvature by R.
(iv) Distance measured in the direction of incident ray are
taken as positive while in the direction opposite of
incident ray are taken negative.
p
p
Real inverted, very small [m < < -1], at F
t-
Real, inverted, diminished (m < -1)
between F and C
888 Chapter 23 Ray Optics
3. AtC
4. Between F and C
5. Atf
6. Between F and P
Table 23.2 Image Formation by Convex Mirror
1. At infinity
c
\
M'
)
2. In front of mirror
M'
M
M'
M
M'
c
Real, inverted, equal in size [m = -1] at C
Real, inverted and very large (m > -1)
between 2F and iiliini.ty
\
Real, inverted, very large [m ~ ( -oo)] at
infinity
Virtual, erect, large in size (m > + 1)
behind the mirror
Virtual, erect, very small (0 < m < < + 1) at F
Virtual, erect, diminished (m < + 1) between
PandF
/
I
I
\
Formula and Magnification for Spherical Mirrors
Mirror fonnula
1 1 1
-=-+-
! v u
where symbol possess their usual meanings.
Lateral magnification
I v f f - v
m=-=-=--=--
0 u f-u f
where I = size of image perpendicular to principal axis
0 = size of object perpendicular to principal ruds.
Axial magnification
2 - "
max = - = :: = ( f J = ( f r
Areal magnification
m = Ar =::.:_=(-! )
2
=(f - v)
2
ar Ao u2 f- u . f
where, A
1
= area of image
A
0
= area of object.
Instance 4 An object of length 2.5 em is placed at l.5f from a
concave mirror; where f is the focal length of the mirror. The length of
the object is perpendicular to the principal ads. Find the length of the
image. Is the image erect or inverted?
(a) -5 em (b) 5 em
(c) 6 em (d, --6 em
Interpret The focal length F = -f
and u = -l.5f
1
1--f

0 F
1----- 1.5 f
Chapter 23 Ray Optics 889
we have,
1 1 1 1 1 1
-+-=-or--+-=--
u v F -l.Sf v f
1 1 1 1
-=----=--
v l.Sf f 3f
Now, ni = _ _l_[_ = -2
u l.Sf
h
or __ = -2 or h
2
= -2h
1
= -5 em.
hl
The image is 5 em long. The minus sign shows that it is inverted.
Instance 5 A concave mirror of focal length 10 em and a convex
mirror of focal length 15 em are placed facing each other 40 em apart.
A point object isplaced between the mirrors, on their common axis and
15 em from the concave mirror. Find the position of the image produced
by the successive reflections, first at concave mirror and then at convex
mirror.
(a) 6 em (b) +10 em
(c) + 15 em (d) +30 (.7n
Interpret According to given problem, for concave mirror.
So,
f= -10 M
u = 15 em andf = - 10 em
.!.+-
1
-=-
1
- ie, v = -30em
v -15 -10
ie, concave mirror will form real, inverted and enlarged
image 1
1
of object 0 at a distanci! 30 em from it, ie, at a distance
40-30 = 10 em from convex mirror.
For convex mirror the image 1
1
will act as an object and so for it
u = -10 em andf = + 15 em.
.!.+-
1
-=_.!._ ie,v= +6Cin
v -10 15
So, final image 1
2
is formed at a distance 6 em behind the convex
mirror and is virtual as shown in figure.
lntext
,
(i); Does the mirror fommla hold g?od for a plane mirrgr? . , , . .. _._. __ _. :,: , .....
(iit An object :is'placed between two plane parallel mirrors. Why do tpe t images fainter .
(iii)' Why are inir:rors used in search-lights parabolic and not concave spherical? , ; )>
If were driving a car, what ofmirtoi would .you prefer to use .at ""''"' '"Adr?
w> Refraction
When light passes from one medium, say air, to another
medium, say glass, a part is refl ected back into the first medium
. : .. .. ;. n-''.': _,: ;\;)assesintu
the second medium, it either bends towards the normal or away
from the normal. This phenomenon is known as refraction.
laws of Refraction (Snell's law)
(i) If meditim 1 is a vacuum (or in practice air) we refer
1
J.l
2
as the absol ute rE>frai'! h" ir><f : )f medium 2 and denote
it by J.l / or simply J.l (if no other medium is there).
(ii) Now, we can write Snell's law as,
J.l sin i = constant
For two media, 11
1
sin i
1
= J.lz sin i
2
... (i)
... (ii)
890 Chapter 23 Ray Optics
2
(iii) Snell's can be written as.
sin i
1
v
1
A.
1
l-12
1112 =-.-.-=-=-=-
Sllll2 v
2
A.
2
11
1
Here, v
1
is the speed of light in medium 1 and v
2
in
medium 2. Similarly 1..
1
and A-
2
are the corresponding
wavelengths.
;1 > ;2
V2 < VI
l-12 > l-11
"-2 < "-1
Rarer
Denser
2
ll :> < u l
"-2 > "-1
Denser
Rarer
If 1-1-
2
::> 1-1-
1
then v
1
> v
2
and A-
1
> 1..
2
, ie, in a rarer
speed and hence, wavelength of light is more.
(iv) In general, speed of light in any medium is less than its
speed in vacuum. It is convenient to define refractive
index 11- of a medium as.
Speed of light in vacuum c
11
= Speed of light in medium v
Instance 6 Light is incident from air on oil at an angle of30. After
moving through oil-1, oil-2, and glass it enters water. If the refractive
indices of glass and water are 1.5 and 1.3, respectively, find the angle
which the ray makes with normal in water.
Air
Oil-1
Oil-2
0 -1 ( 1 J sm -
2.6
(b) sin-
1
(_]__J
2.6
(a)
(c) sin-
1
(-
1
J
3.6
Interpret As we know
(d) sin-
1
(2.6)
1-l sin i = (constant)
=> !l-air sin = 1-l-glass sin r (glass)
0 0 !l-air 0 0
=--Sin lair
1-l-glass
Again, llglass sin iglass = llwater sin r water
From Eqs. (i) and (ii)
sin 30 = 1.3 sin r
0 1 1
smr=--=-,
, 2xl.3 2.6
r=.sin-
1
(-
1
J
2.6
... (i)
... (ii)
Instance 7 A ray of light is incident on a transparent glass slab of
refractive index 1.62. If the reflected and refracted rays are mutually
perpendicular, what is the angle of incidence?
(a) 58.3 (b) 85.3
(c) 60 (d) 65
Interpret Let the angle of incidence, angle of reflection and an,le
of refraction be i , rand r', respectively.
Now, as per the question 90 -r + 90- r' = 90
=> r' = 90- i (because i = r)
In case of reflection according to Snell's law, 1 sin i = 1-l sin r '
or sin i = 1-l sin (90 - i)
=> tan i = 1-l
or i = tan-
1
[!1-] = tan-
1
(1.62) = 58.3
Instance 8 Refractive index of glass with respect to water is 1.125.
If the absolute refractive index of glass is 1.5, find ,the absolute index
of water.
(a) 1.33 (b) 2.33
(c) 0.33 (d) 0.44
Interpret Here, the refractive index of glass with respect to water
ie, wll-g = 1.125 and absolute refractive index of glass llg = 1.5.
We know that
Apparent Shift of an Object due to Refraction
Due to bending of light at the interface of two different
media, the image formation due to refraction creates an illusion
of shifting of the object position.
Consider an object 0 in medium. After refraction, the ray
at the interface bends. The bent ray when it falls on our eyes, is
perceived as corning from I.
For nearly normal incident rays, 8
1
and 8
2
will be very small.
Rarer
J.lz
. AB
tan 8
1
= sm 8
1
= -=-':-:-:-':__-:-:----:---.-=-:-----"':'--'---:----:--
Object distance from the refracting surface
Similarly,
. AB
sm8
2
= -------------,-------
Image distance from the refracting surface
sin 8
1
_
11
_ 11
2
:::} AB
1
AB _ 11
2
sin8
2
-
1 2
- 11
1
OB BI - 11
1
BI Apparent depth J..lz
-= =-
OB Real depth J..L
1
So, Shift = Real depth- Apparent depth = Real depth ( 1- J
Case I
If ).ll < ).lz
Shift becomes negative, image distance > object distance,
ieJ image is farther from the refracting surface.
Case II
If ).ll > ).lz.
Shift becomes positive, image distance < object distance,
ie image is closer to the refracting surface.
Case III
If ).lz = 1 or ).!
1
).l
Shift = Real depth ( 1 - J
Instance 9 A fish in an aquarium, approaches the left wall at
a rate of 3 ms-1, and observes a fly approaching it at 8

If the
refractive index of water is (4/ 3), find the actual velodty of the fly.

).lX
(a) 3.75 ms-
1
(b) 2.75ms-
1
(c) 0.75 ms-
1
(d) 4.75 ms-
1
Interpret For the fish, appa.rent distance of the fly from the wall
of the aquarium is ).lX. If x is actual distance, then apparent
velocity will be d(J.!X)
dt
(v. pp)fly = !l vfly
Now, the fish observes the velocity of the fly to be 8 ms-
1

Therefore, apparent relative velocity = 8 ms-
1
Vfish + Cvapp)fly = 8 ms-
1
3 + ).l vfly = 8
vf!y = 5 x = 3.75ms-
1
4
Instance 10 A layer of oil 3 em thick is flowing on a layer of
coloured water 5 em thick. Refractive index of coloured water is 5/3
and the apparent depth of the two liquids appears to be 36/7 em. What
is the refractive index of oil?
(a) 1.4
(c) 3
Interpret
or
or
(b) 2.4
(d) 2
Apparent depth (AD) = .EL + 2_
Ill ' 112
,
36 5 3
-=--+-
7 5/3 J..lz
.2_= 36 -3= 15
J..lz 7 7
7
J..lz =- = 1.4
5
Chapter 23 Ray Optics 891
Total Internal Reflection
Whenever a ray of light goes from a denser medium to a rarer
medium it bends away from the normal. As angle of incidence
in denser medium increases, angle of refraction also increases in
rarer medium.
The angle of incidence in denser medium for whi.ch the
angle of refraction in rarer medium is 90 is called the critical
angle (C) .
=>
=>
sin C = =
sin 90 J..ldenser J..ld

J..ld
c = sin-
1

Now, if the angle of incidence in the rarer medium is greater
than the critical angle (C), then the ray instead of suffering
refraction is reflected back in the same (denser) medium.
This phenomenon is called total internal reflection. For
total internal reflection to take place following set of conditions
must be obeyed.
(i) The ray must travel from denser medium to rarer
medium.
(ii) The angle of incidence i must be greater than critical
angle C.
Instance 11 An isotropic point source (bulb) is placed at a depth h
below the water surface. A floating opaque disc is placed on the surface
of water, so that the bulb is not visible from the surface. What is the
minimum radius of the disc?
Take refractive index ofwater=).l.
Interpret As shown in figure, light from bulb will not emerge out
of the water if at the edge of disc.
1
i>C
sin i >sin C ... (i)
Now, if R is the radius of disc and h is the depth of bulb from it
. . R
smt=-,===
Jiz +hz
and
. c 1
sm =-
J..l
So, Eq. (i) becomes
or
R 1
--===>-
+hz J..l
h
R>--

892 Chapter 23 Ray Optics
Refraction from a Spherical Surface
Spherical surfaces are of two types
(i) Convex
(ii) Concave
()
1
~ I 2
;;, :
1E'
0 I 2
-- -p ---------- ------ -- --- ---
For both surfaces refraction formula is given by
~ - ~ = Jllz-1
v u R
1
1-1
2
is refractive index of second medium with respect to first.
If 1-1
1
and 1-lz are refractive indices of first and second medium
with respect to air, then,
llz _ J-11 = 1-lz 1-11
v u R
instance 12 A linear object of length 4 em is placed at 30 em
from the plane surface of hemispherical glass of radius 10 em. The
hemispherical glass is surrounded by water. Find the final position and
size of the image:
(a) 5.3 em
(c) 5 em
(b) 4.3 em
(d) 2.3 em
;nterpret
4 3
For 1" surface 1-1
1
= -,J-1
2
=- ,u = -20cm,
3 2
and R = +lOcm,
8"
5.3cm
A"
1----- v' - - - - ~
Using
1-lz _ J-11 = (J.lz -1-11)
v u R
==>
(3/ 2) - ( 4/ 3) = (3/ 2- 4/ 3)
v (-20) 10
==> v = -30 em
Using A' B' = => A' B' = ( 4/ 3) ( - 30)
AB JlzU ( 4cm) (3/ 2) ( -20)
==> A'B' = 5.3 em
A'B' behaves as the object for plane surface
3 4 '
Jl
1
=
2
,Jlz=j and R=oo,u=-40
==>
llz_ = llz
v' u'
==>
( 4/ 3) = (3/ 2)
v' ( -40)
Solving it we will get, v' = -35.4cm
N
. A' B'' (Jl
1
v')
ow usmg --= --
, A' B' (Jl
2
u')
A"B" = (3/ 2)(-35.4) => A"B" = 5.3cm
(5 .3) ( 4/ 3)( -40)
=>
The final images in all the above cases are shown in figure .
~ ' ~ Lens
Lens is a transparent medium bounded by two curved
surfaces.
Lenses are of two types
1. Convex or convergent lens
2. Concave or divergent lens
1. Convex or Convergent Lens
The traqsparent medium bounded by two bulging surfaces
is called convex lens. It is ofthree types (as shown) .
(a) Double-convex
lens
(b) Plano-convex (c) Concavo-convex
lens lens
2. Concave or Divergent Lens
The transparent medium bounded by two hollow surfaces is
called concave lens. It is of three types (as shown).
(a) Double-concave (b) Plano-concave (c) Concavo-concave
lens lens lens
Some Definitions Relating Lenses
Optical centre The optical centre is a point within or
outside the lens, at which incident rays refract without deviation
in its path.
s
Principal axis The straight line passing through the optical
centre of lens is called principal axis of lens.
1
Principal focus Lens has two principal foci .
(i) First principal focus It is a point on the principal axis
of lens, the rays starting from which (convex lens) or appear to
converge at which (concave lens) become parallel to principal
axis after refraction.
. ..
... -::
.. _... F1
(ii) Second principal ,focus It is the point on the principal
axis at which the rays coming parallel to the principal axis
converge (convex lens) or appear to diverge (concave lens) after
refraction from the lens.
Both the foci of convex lens are real while that of concave
lens are virtual.
Focal length The distance between focus and optical
centre of lens is called focal length of lens.
Table 23.3 Formation of Image by a Convex Lens
1. At infinity
2. Beyond 2F
1
Chapter 23 Ray Optics
l,i
Laws of Formation of Images by Lens '
'
(i) The rays corning parallel to principal axis of lens
through the focus after refraction.
(ii) The rays corning from the focus ofltilS go parallel
principal of lens after refraction. ':
(iii) The rays of light passing through optical centre go
straight after refraction without changing their path.
Lens Maker's Formula
If R
1
and R
2
are the radii of curvature of first and seconq
refracting surfaces of a thin lens with optical centre C of
length/ and refractive index


then according to Lens Maker'$
formula
c
u-----1...._--v
_ _..!__]
f Rl R2
..!..=(!l-l)(_..!__ _ _..!__J
f R1 R2
where,


= is refractive index of material of lens with'
respect to surrounding medium. .
Thin lens formula is
1 1 1
-=- --
! v u
At the principal focus (F
2
) or
in the focal plane
Between F
2
and 2F
2
Real, inverted and extremely
diminished
Real, inverted and diminished
894 Chapter 23 Ray Optics
4. Between F
1
and
2Fl
5. At F
1
6. Between F
1
and
optical centre
Formation of Image by Concave Lens
The image formed is always virtual, erect and diminished
and lies between the lens and F
2
for all positions of the
object.
Instance 13 The focal length of convex lens is 10 em in air. Find its
focal length in water. (Given, llg. = 3/ 2 and llw = 4/ 3)
(a) 10 em (b) 20 em
(c) 30 em (d) 40 em
Interpret -
1
-=CI!g -1)(2__2_J ... (i)
fair Rl Rz
and 1 (llg X 1 1 J
ff'ater = ll w -
1
Rl - Ri
.. (ii)
Beyond 2F
2
At infinity
On the same side as the
object
Real, inverted and of same size
as the object
Real, inverted and magnified
Real, inverted and highly
magnified .
Virtual, erect and magnified
Dividing Eq. (i) by (ii), we get
fwater ( llg -1 J
"fair = llg f ll w -1
Substituting the values,
(3/ 2-1)
fwater = (
312
)fair
---1
4/ 3
= 4 fair
= 4 X 10
= 40cm
Instance 14 An object is placed at a distance of 10 em to the left
on the axis of a convex lens L
1
of focal length 20 em. A second convex
lens L
2
of focal length 10 em is placed co-axially to the right of the lens
L
1
at a distance of 5 em from it. Find the position of the final image and
its magnification.
(a) 163. em on the right of the second lens, 3.33
3 .
(b) 163. em on the right of the second lens, 1.33
3
(c) 163. em on the right of the first lens, 1.33
3
(d) None of the above
Interpret Here, for 1" lens,
=>
=>
u
1
= -10 em
f
1
= 20 em
1 1 1
---=-
vl u1 !1
1 1 1
-=---
vl
20 10
v
1
=-20cm
L1 L2
~
o , o;:;; 0
2
'! o
\ '! !
';.: \ i
\! v
,1,. 1'2
Scm
ie, the image is virtual and hence lies on the same side of the
object. This will behave as an object for the second lens.
1 1 1
For 2nd lens, ---=-
Vz u
2
fz
Here, u
2
= -(20 + S),J
2
= 10cm
1 1 1 50 2
-+-=- => v
2
=-=16-cm
v
2
25 10 3 3
ie, final image is at a distance o; 163_ em on the right of the
second lens. , 3
T)le magnification of the image is given by;
m = 2:1_ Vz = 20 _29_ = _ = 1.33
u
1
u
2
10 3x25 3
Magnification of Lens
The lateral, transverse or linear magnificauon produced by
a lens is defined by
Height of image I
m= =-
Height of object 0
A real image II' of an object 00' formed by a convex lens is
shown in figure.
Height of image = _!__ = : ' ' ''
Height of object 00' u
Substituting v and u with proper sign, ) ,; .
. If -I v
'.
00' 0 -u
I v
or -=m=-
0 u
Thus,
v
,
m=-
u
Chapter 23 Ray Optics 895
Important Features
1. Power of lens P = --
1
-
f(inm)
=>
P= 100
j(incm)
Power of convex lens is positive and of concave lens is
negative.
2. If distance of an object from first focus of lens is a
1
and
distance of image from second focus is a
2
, then its focal
length.
f = ~ a l a 2 .
This is Newton's formula.
3. If two or more lenses are placed in contact, then
equivalent focal length of the combination.
1 1 1 n 1
-=-+-+ .. = L.-
f !1 fz i=l h
Power of combination
n
p =Pl +Pz + ... = L,P;
i=1
Magnification of combination
n
M = m
1
xm
2
x .. . =11m
i=l
4. If two lenses offocallengthsf
1
andf
2
are separated by a
distance x, then its equivalent focal length
1 1 1 X
-=-+- --
F !1 fz fdz
Power of combination,
P = P1 + Pz- x P1P2
Total magnification remains unchanged ie,
m=m
1
xm
2
5. If a lens is made of a number
of layers of different refractive
indices, then number of images
of an object formed by the lens
is equal to number of different
media.
6. Cutting of a lens
(i) If a symmetrical convex
lens of focal length f is
cut into two parts along
its optic axis, then focal length of each part (a
plano convex lens) is 2f. However, if the two parts
are joined as shown in figure, the focal length of
combination is again f.
2(, 2f f f
(a) (b) (c) (d)
I
896 Chapter 23 Ray Optics
(ii) If a symmetrical convex lens of focal length f is cut
into two parts along the principal axis, then focal
length of each part remains changed at f . If these two
parts are joined with curved ends on one side focal
length of the combination is f_ . But on joining two
2
parts in opposite sense the net focal length becomes ~
(or net power = 0).
(a) (b) (c) (d)
Table 23.4 Difference between Lens and Mirror
1. Convex lens + ve + ve
2. Concave mirror - ve + ve
3. Concave lens - ve -ve
4. Convex mirror + ve -ve
,
7. Silvering of a lens
(i) Let a plano-convex lens is having a curved surface of
radius of curvature R and has refractive index J.l if
its plane surface is silvered, it behaves as a concave
mirror of focal length.
R
f =- 2 ( ~ - t - 1 )
(ii) If the curved surface of plano-convex lens is silvered
then it behaves as a concave mirror of focal length.
R
f =- 2!l
(iii) If one surface of a symmetrical double convex lens
(R
1
= R
2
= R) is silvered, then the lens behaves as a
concave mirror of focal length
f = R
2(2J.l-l)
8. The tabular difference between lens and mirror is given
in table.
converging
converging
diverging
diverging
Instance 15 A convergent lens of 6 D is combined with
a diverging lens of -2 D. Find the power and focal length of the
combination.
(a) 26 em
(c) 30 em
(b) 20 em
(d) 25 em
Interpret Here, P
1
= 6 D, P
2
= -2 D
Using the formula, P = P
1
+ P
2
= 6- 2 = 4 D
f = liP= 1/4 m = 25 em.
Is the ratio of frequencies o_f ultraviolet rays and ....... ,,a""'
Can convergent
Why
!I) Refraction Through a Prism
A prism is a homogeneous,
transparent medium bounded
by two plane surfaces inclined
at an angle A with each other.
These surfaces are called as
refracting surfaces and the
angle between them is called
angle ofprismA.
p
A
Figure shows the 0 R
refraction of monochromatic
light through a prism. Here' i and e represent the angle of
incidence and angle of emergence respectively, r
1
and r
2
are two
angles of refraction. If J.l is the refractive index of the material of
the prism, then
sini sine
J.l=--=--
sinrl sinr
2
The angle between the incident ray and the emergent ray
is lmown as the angle of deviation o. For refraction through a
prism it is found that
i + e =A + o and r
1
+ r
2
= A
Minimum Deviation
It is found that the angle
of deviation o varies with
the angle of incidence i of
the ray incident on the first
refracting face of the prism.
The variation is shown in Om
figure and for one angle of
incidence it has a minimum
value omm. At this value
i = e
It therefore, follows that
A
r=-
2

r1 = r2
Further at o
1
= om;= (i + 0 -A
Chapter 23 Ray Optics 897
Instance 16 A convex lens of 10 em focollength is combined with a
concave lens of 6 em focollength. Find the focollength of the combination.
(a) -15 em (b) 15 em
(c) 10 em (d) -10 em
Interpret Here,f
1
= 10 cm,f
2
= -6 em, F =?
Use the formula .!. = ..!._+..!._ = _.!__.!_ = _ _.!_
' F !
1
!
2
10 6 15
or
F = -15 em
. A+om
!=--
2
sm --
[A+om)
smt 2
J.!=-.- or J.l= A
smr sin-
2
For thin prism, om" {ll l)A.
<Jl T
Instance 17 The angle of minimum deviation for a glass prism
with J.l = .J3 equals the refracting angle of the prism. What is the angle
of the prism?
Interpret
Using,
we have
or
2
. A A
sinA Slll - COS -
.J3=--= 2 2
sin(1) sin(1)
A .J3
COS-=-
2 2
A
- = 30 or A = 60
2 .
.Dispersion
ut
Dispersion of light is the phenomenon of splitting of white
light into its constituent wavelengths on passing through a
dispersive medium, eg, prism. Cause of dispersion is the variation
of refractive index of prism with wavelengJ:h. As A.v < A.R, hence,
J.lv > llR and consequently Ov > Ow
898 Chapter 23 Ray Optics
Angular Dispersion It is the angular separation between
the fwo extreme rays.
Angular dispersion 8 = 8v - 8R
= C11v - 11R )A
Dispersive Power The dispersive power of a prism
material is measured by the ratio of angular dispersion to the
mean deviation suffered by light beam.
:. Dispersive power
ro- Ov -oR _ 1-lv -1-lR

where 11 is the mean value of refractive index of prism.
The dispersive power of a prism depends only on its material
and is independent of angle of prism, angle of incidence or size
of the prism.
Dispersive power is a unitless and dimensionless term.
Dispersive power of a flint glass prism is more than that of
a crown glass.
Dispersion without Deviation (Direct Vision Prism)
1. To produce dispersion mean deviation we use a
combination of two prisms of different materials such that
A'= -(l-l.-
1
JA
!-l-1
2. Net dispersion caused
= C11v- 11R) A + C11' v - 11' R)A'
= (11- 1)A (m - m') = 8 (m - m')
Deviation without Dispersion (Achromatic Prism)
1. To produce deviation
without dispersion
we use a combination
of two prisms of
different materiaPs
such that
A'=- [!-lv -1-lR] A

2. Resultant produced =O[ 1-:]
Instance 18 Find the dispersion produced by a thin prism of 18
having refracting index for red light = 1.56 and refractive index for
violet light = 1.68.
(a) 2.16
(c) 3.16
(b) 1.16-
(d) 2.10
Interpret We know that dispersion produced by a thin prism
e = C!-l v -11R)A
Here, 1-lv = = 1.56 and A = 18
e.= (1.68 -1.56) x 18 = 2.16
Instance 19 Calculate the dispersive power for crown glass from
the given data
(a) 0.01639
(c) 0.05639
1-lv = 1.523 and 1-LR = 1.5145
(b) 1.05639
(d) 2.05639
Interpret Here, 1-lv = 1.523 and 1-lR = 1.14S.
M fr
. . d l.S23+1.S145
ean re actlve m ex, 11 = l.S187S
2
Dispersive power ro is given by,
ro = !lv -llR = l.S23 -l.S14S
0
_
01639
(!l-1) (l.S187S-1)
Instance 20 A prism of crown glass with refracting angle of so
and mean refractive index = 1.51 is combined with a fiint glass prism
of refractive index = 1.6S to produce no deviation. Find the angle of
fiint glass.
(a) 3.92 (b) 4.68
(c) 5.32 (d) 7.28
Interpret Let A' be the angle of flint glass prism.
Here, A = so and J-l = 1.51 for crown glass prism.
8 = (!l-1)A= (l.S1-1)xS = 2.SS
0
Deviation produced by flint glass
o' = (!!' - 1)A' = (1.6S -1)A' = 0.6SA'
For no deviation, 8' = 8 or 0.65A' = 2.55
> Optical Instruments
A'= 2.S5 = 3.92o
0.65
Optical instrument is a device which is made from proper
combination of mirrors, prisms and lenses. The principle of
working of optical instruments depends on laws of reflection and
refraction of light.
Microscope
It is an optical instrument which forms a magnified image
of a small nearby object and thus, increases the visual angle
subtended by the image at the eye so that the object is seen to be
bigger and distinct.
) .. l
(i) Simple microscope A simple microscope is a convex
lens of short focal length which is fixed in a frame provided with
handle. . cj ',
Magnification of simple microscope
(a) When final image is formed at least distance of
distinct vision,
D
M=1+-
f
(b) For relaxed eye, M = D
f
where D = least distance of distinct vision.
(ii) Compound microscope Figure shows a simplified
version of a compound microscope. It consists of two converging
lenses arranged coaxially. The one facing the object is called
objective and the one close to eye is called eye piece. The
objective has a smaller aperture and smaller focal length than
those of the eye piece.
h
Magnification of compound microscope
(a) For relaxed eye
= _ vo (E-J
Uo fe
In this position, length of microscope
= Vo + fe
(b) When final image is formed at least distance of distinct
VlSlOn.
MD DJ
Uo f e
Length of microscope,
L
0
=v
0
+Ue
Telescope
v
0
= distance of first image from object lens.
U
0
= distance of object from objective lens.
f. = focal length of eye piece.
Telescope is an optical instrument which increases, the
visual angle at the eye by forming the image of a distant object
at the least distance of distinct vision, so that the object is seen
distinct and bigger.
(i) Astronomical telescope It consists of two converging
lenses placed coaxially. The one facing the distant object is called
the objective and has a large aperture and large focal length.
The other is called the eye-piece, as the eye is placed closed to
it. The eye-piece tube can slide within the objective tube, so that
the separation between the objective and the eye-piece may be
varied.
'
Chapter 23 Ray Optics 899
Object
Magnification of astronomical telescope
(a) For relaxed eye, M = _ fo
fe
In this position, length of telescope

(b) When final image is formed at least distance of
distinct vision
M = _ f o ( 1 + fe J
D fe D
Length of telescope
LD =fo+ue
f o = focal length of objective lens
f e = focal length of eye piece
(ii) Terrestrial telescope In an astronomical telescope,
the final image is inverted with respect to the object. To remove
this difficulty, a convex lens of focallengthf is included between
the objective and the eye-piece in such a way that the focal plane
of the objective is a distance 2f away from this lens.
B

A


Magnification of terrestrial telescope
(a) For relaxed eye, = 1
In this position, length of telescope
= f o + 4f + f e
(b) When final image is formed at least distance of distinct
vision,
M = f o (1+ fe J
D fe D .
Length of telescope, n
LD = fo + 4 f +de
f o = focal length of objective lens
f e = focal length of eye peice
Galilean telescope A simple model of Galilean telescope
is shown in figure. A convergent lens is used as the objective and
a divergent lens as the eye-piece.
900 Chapter 23 * Ray Optics
fo ______ ___..,
8
1
A
Magnification of Galilean telescope
(a) For relaxed eye, = fo
fe
In this position, length of telescope
L, = fo- fe
(b) When final image is formed at least distance of distinct
vision M = fa (1 -fe J
' D fe D
Length of telescope
Lv = fo -u,
Instance 21 An object is seen through a simple microscope of focal
length 20 em. Find the angular magnification produced if the image is
formed at 30 em from the lens.
(a) 2.08
(c) 3.08
Interpret Given,f = + 20 em
(b) 2.05
(d) 1.5
and v = - 30 em ,
Using thefonnula, .!._.!.=.!. wehave,
v u f
1 1 1
-30 -U
0
20
u
0
= 12 em
'{<The >angular magnification, M = .!!_ =
25
= 2.08
uo 12
Instance 22 A galilean telescope is 27 em long when focussed to
form an image at infinity. If the objective has a focal length of 30 em,
what is the focal length of the eye piece?
(a) 3 em (b) -3 em
(c) 2 em (d) -2 em
Interpret Given,fo = + 30 em. Objective
Length of telescope is given
27cm.
Therefore, u, = + 3 em.
For the final image at infinity,
the intermediate image should lie
Eye-piece
at first focus of eye piece of the I-- 27 em
Galilean telescope. 30 em I
fe=-3cm
Resolving Power of a Microscope
Resolving p;Jwer of e. microscope is defined as the reciprocal
of the least separation between two close objects, so that they
appear just separated, when seen through the rnicroscope.
The least separation between two objects, so that they
appear just separated is given by
d=--"'-
2J.1sin8
where fl is che refractive index of the medium between the
objective of the microscope and the object. This distance is called
limit of of the microscope.
Resolving power of a micro5mpe = .!_ =
2
J.1 sine
d lc
8 = half angle of the cone of light from the point object,
fl sin 8 = numerical aperture
.... de. _____ _____ _
0
1. Resolving power of microscope increases with increase in the value
of the refractive index of the meaium between objective and object
that's why oil immersion objective microscopes are used to achieve
high resolving power.
2. The resolving power of microscope increases, with decrease in the
value of the wavelength of the light used to illuminate the object,
so microscopes using light for illu(Tlinating the objects
are used to achieve high resolving power. These are called ultra
microscopes.
Higher resolving power is obtained in electron microscope.
> Resolving Power of Telescope
Resolving power of telescope is defined as the reciprocal
of the smallest angular separation between two distant objects,
so that they appear just separated, when seen through the
telescope.
The smallest angular separation between two objects, so
that they appear just separated is found to be
de= 1.
2
21c
D
where D is the diameter of objective
D
Resolving power of telescope
1.22/c
lntext Questions 23.3
en does a ray on _a prism deviate away from the base?
"ngs) observed sometimes round the sun or moon?
f for lights ?f yellow, green and red colours are f!y. llg and flr respectively. Rearrange
_;
e position of a object relative to a biconvex lens so that it behaves like magnifying lens?
erted, it serve as a microscope?
Chapter 23 Ray Optics 901
Chapter Compendium
are o\:1eyed at every reflectin,g surface.
ofsy:mmetrical spherical surfaces, are
two (i) convex, and (ii) concave.
which the reflection takes place at the bulged
a convex mirror and the mirror in which
place at the depressed surface is called a
rays, the refracted ray and the normal
surface separating the two media-all lies in one
law For any two media the ratio of the sine of the
of incidence to the sine of the angle of refraction
constant for a light beam of a particular frequency, ie
T
t

C is the critical angle
,
,,
'
10. Refraction at Sphe,rical Surfaces,
(i) For a spheric1il surface, f.Lz . ..., f.LI = Hz - f.L1
(ii) Magnification, m =
111
::_
flzU
(iii) When the object is
relation can be obtainted vv<<.:muL)(.UI
relation becomes
_ f.L2 = !11 -J.Lz
u v R
11. Thin Lens
1 1 1
12. Thin lens fonnula .,--- =-
v u f
13. Linear magnification of a lens
14. Power ofLens
(i) Power of a lens = (1/J) where f is the focal
the lens in meters with proper sign . ..
(ii) Power of combination of lenses.P ':"
where P
1
, P
2
, P
3
, are powers o{co
(iii) Magnification produced by equiValent lens.uf = ;5 ,m
2
15. Prism
(i) Refracting angle
(ii) A + o = i + e
902 Chapter 23 Ray Optics
" I
s ..
l '
l ~ "l
,
I
Illustrative
Example 1 A beaker contammg liquid is placed on a table
underneath a microscope which can be moved along a vertical scale.
The microscope is focussed, through the liquid, on a mark on the
table and the reading on the scale is a. It is next focussed on the upper
surface of the liquid and the reading is b. More liquid is added and the
observations are repeated, the corresponding readings being c and d.
The refractive index of the liquid is
d-b
(a)
d-c-b+a
b-d
(b)
d -c-b+a
d-c-b+a
(c)
d-b
(d)
d+c-b-a d-b
Solution The real depth = (Refractive index) apparent depth
::::>In first case, the real depth h
1
= ).!(b- a).
Similarly, in the second case, the real depth h
2
= J.!(d- c)
Since, h
2
~ h
1
the difference of the depths = h
2
- h
1
= J.!(d- c- b + a)
Since, the liquid is added in second case, h
2
- h
1
= d - b
d-b
ll= d-c-b+a
Example 2 A ray of light from a denser medium strikes a rarer
medium at angle of incidence i. The reflected and refracted rays make
an angle of90 with each other. The angles of reflection and refraction
are r and r', respectively. The critical angle is
(a) sin-
1
(tan r)
(b) sin-
1
(cot i)
(c) tan-
1
(sin r)
(d) tan-
1
(sin i)
Solution Applying Snell's law for refraction,
sini = 11
2
sinr' 11
1
From the given condition, r + r' = 90
sin r' =cos r
Solution of Eqs. (i) and (ii) yields, sin i =
112
cosr 11
1
According to the Law for refraction
i = r
,
... (i)
... (ii)
.. . (iii)
... (iv)
/.12
Using Eqs.(iii) and (iv), we obtain
sini = 11
2
cosi 11
1
tani = 1-lz
Ill
Since, at the time of total internal reflection,
sinec = 1-lz, using Eq.(v) we obtain ec = sin-
1
(tanr) .
Ill
... (v)
Example 3 A given ray of light suffers minimum deviation in an
equilateral prism R Additional prism Q and R of identical shape and of
the same material as P are now added as shown in the figure. The ray
will now suffer
(a) greater deviations (b) no deviation
(c) same deviation as before (d) total internal reflection
Solution No deviations occur on interfaces 2 and 3 as there is no
change in medium. However, deviation at interface 4 is same as
it was on interface 2 with only prism P.
Example 4 The sun (diameter D) subtends an angle of e rad
at the pole of a concave mirror of focal length f The diameter of the
image of the sun formed by the mirror is
(a) f6 (b) f26!D
(c) 2f6 (d) De
Solution Since, the sun is at very large distance, u = oo
904 Chapter 23 Ray Optics
1 1 1
~ -+-=-
00 v f
==> v=f
If the diameter of the image be d,
T
d
2
.J_
d/2
--=a:::> d = (2a)v
v
:::> d=(2a)v
Putting, 2a =6 and v = f, we obtain d = f6
Example 5 A ray enters a glass sphere of refractive index fl = ../3 at
an angle of incidence 60 a ray is reflected and refracted at the farther
surface of the sphere. The angle between the reflected and refracted rays
at this surface is
(a) 50
(c) 90
(b) 60
(d) 40
Solution Refraction at P
sin 60 = ../3 .
sinr
1
sinr
1
= (1/2) ==> r
1
= 30
.l Since
r2 = r;
r
2
= 30
sinr
2
1
Refraction at Q = ----
sini
2
- ../3
Putting r
2
= 30, we obtain i
2
= 60
Reflection at Q, r; = r
2
= 30
Example 6 A soldier directs a laser beam on an enemy by reflecting
the beam from a mirror. If the mirror is rotated by an angle 8, by what
angle will reflected beam rotate?
(a) 8/2 (b) 8
(c) 28 (d) None of these
Solution Let MPM
2
be the initial position of the mirror.
The mirror is rotated through an angle 8 to the position
M't OM2.PO is the incident light. OQ on the initial reflected
ray and OQ' is the reflected ray after rotating the mirror by
angle 8. If i = initial incidence angle, then L.POQ = 2i and
L.POQ' = L.PON' +L.N'OQ = 2i- 26.
Q'
p
Q
:. The r e f l e c t e ~ beam rotates through an angle 28.
Example 7 When an object is at distances of u
1
and u
2
from the
poles of a concave mirror, images of the same size are formed. The focal
length of the mirror is
(a) lu1 +uzl
(b) lu1 -uzl
(r) lul ;uzl
(d) lu
1
;u
2
1
Solution One image will be real and the other will be virtual.
___ Since they are of the same size, one will have magnification m
and the other -m.
1 1 1
-+-=-
ul u
1
m f
or 2_(1 + ]:_) = _!_
_ u
1
m f
... (i)
1 1 1
----=- and
Uz u
2
m f
2_(1-]:_)=.!.
u
2
m f
... (ii) or
From Eqs. (i) and (ii), we get
ul +uz = 2
f
f = ul +uz
2
or
Example 8 A ray of light passes through four transparent media
with refractive indices J..lJJ J..lz, J..t
3
and J..l
4
as shown in the adjacent figure.
The surfaces of all media are parallel. If the emergent ray CD is parallel
to the incident ray AB, we must have
(a) J..l1 = J..l3 (b) J..l2 = J..l4
(c) J..l4 = J..l1 (d) J..l2 = J..l3
Solution Considering Snell's law, J..l sin 8 = constant, and
i
1
= 4 (given).
Since,
sin i
1
= flz sm 1
2
sin i
2
11
1
' sin i
3
= 113 sini3 = 114
11
2
'-sin i
4
11
3
sini1 = 11
4
sin i
4
11
1
i
1
= i
4
, therefore J..t
4
= J..t
1

Example 9 Light is incident normally on face AB of a prisTTJ. as
shown in the figure. A liquid of refractive index J.1 is placed on face AC
of the prism. The prism is made of glass of refractive index 3/2.
Tile limits of fl for which total internal reflection takes place on
faceAC is
3
(a) !J.> -
4
8
3J3
(b) ll <--
4
(d) .)3
!J.<2
Solution
Critical angle between glass and liquid face is
sine =
312

c ll 21J.
Angle of incidence at face AC is 60
ie, i = 60
For total internal reflection to take place
... (i)
i > ec -----------------------
----- ------------- -----
or sin i >sin ec _-:_-_:::.-_-_-_:::::.-_:::.-_-_-_:::.-_-:_-_:
. 600 3
or sm >--
or
or
2}l
.)3 3
->-
2 2}l
}l> .)3
Example 10 Two identical glass (J.1
8
=3!2)
equiconvex lenses of focal length fare kept in contact.
.The space between the two lenses is filled with water
Cflw = 4/3). The focal length of the combination is
(a) f
(c) 4f
3
(b) f_
2
(d) 3f
4
8

Let R be the radius of curvature of each surface. Then
1 ( 1 1 J
7=CL
5
-
1
) "R+"R .
For the water lens
; . =(i-
1
)( -*+*) =M -7 l ;. =-
,
Chapter 23 Ray Optics 905
. 1 1 1 1
Nowusmg, -=-+-+- wehave
Ff1fzf3
1 1 1 1
-=-+-+-
F f f f'
2 2 4
=---=-
! 3f 3f
F= 3f
4
Example U A concave lens jQrms the image of an object such
that the distance berween the object. and image is 10 em ana the
magnification produced is 1/4. The focal (ength of the lens will be
(a) 8.6 em (b) 6.2 em
(c) 10 em (d) 4.4 em
Solution Concave lens fom1S the virtual image of a real object.
So let
u=-4xandv=-x
Then 3x = 10 ern
10
or x =-ern
3
40 10
u=--cmand v=--cm
3 3
S b
. . . 1 -3 3
u stltutmg m -=-+-
! 10 40
or
or f = - 4.4 ern
0
3x
4x
Example 12 A microscope has an objective of focal length 1.5 em
and eye piece of focal length 2.5 em. If the distance between objective
and eye-piece is 25 em, what is the approximate value of magnification
produced for relaxed eye?
(a) 75 (b) 110
(c) 140 (d) 25
Solution Length of the tube is L = V
0
+f
Now applying
V
0
= L- F.= - 2.5 = 22.5 ern
.!.. - .!.. = .!.. we have
Vo Uo fo
1 1 1
----=-
22.5 U
0
1.5
luolz1.6 ern
IMI=vox!!_
uo f.
=(22.5)
\ 1.6 2.5
z140.
Chapter Practice
Exercise I
Photometry
1. What is the ratio of luminous intensity ot two sources,
which produce shadows of equal intensities at ci1stance
25 em and 50 em from the photometer screen?
(a) 1 : 4 (b) 4 : 1
(c) 1 : 2 (d) 2 : 1
2. The time required for making a print a distance of 0. 25 m
from a 60 W lamp is 5 s. If the distance is increased t o
40 em, the time required in second to make a similar print
is
(a) 3.1
(c) 12.8
(b) 8
(d) 16
3. A source is at 4m height above the centre of a circular table
of a circular table of radius 3m. The ratio of illuminance
at 0 and P will be
, S
4m
PF---3-m_...Jo
64
(a) 125
(c) 1
(b) 125
64
(d) 16
25
4. A lamp of 250 candela power is hanging at a distance of
6 m from a wall. The illuminance at a point on t he wall at
a minimum distance from lamp will be
(a) 9.64lux (b) 4.69 lux
(c) 6.94lux (d) None ofthese
5. An electric bulb illuminates a plane surface. The intensity
of illumination on the surface at a point 2 m away from
the bulb 5 x 10-
4
phot (lumen cm-
2
) . The line joining the
bulb to the point makes an angle of 60 with the normal
to the surface. The intensity of the bulb in candela (candle
power) is '
(a) 40 x 10-
4
(c) 40vG
(b) 40
(d) 20
6. A lamp is hanging at a height of 40 em from the centre
of the table. If its height is increased by 10 em, the
illuminance of the lamp will decrease by
,
(a) 10%
(c) 27%
(b) 20%
(d) 36%
7. In a photometer, two sources of light when placed at
30 em and 50 em respectively produce shadows of equal
intensities. Their candle powers are in the ratio of
(a) _.2._ (b) 16
25 25
3 5
(c) - (d)
5 3
8. A book can be read if it is placed at a distance of 50 em
from a source of 1 cd. At what distance should the book
placed if the source is of 16 cd?
(a) 8 m (b) 4 m
(c) 2m (d) 1 m
9. In a grease spot photometer, light from a lamp with dirty
chimney is exactly balanced by a point source distance
10 em from the grease spot. On dearing the dirty chimney,
the point source is moved 2 em to obtain a balance
again. Then the percentage of light absorbed by the dirty
chimney is nearly
(a) 64% (b) 36%
(d) 56% (c) 44%
10. Two point sources A and B of luminous intensities 1 cd
and 16 cd respectively are placed 10.0 em apart. A grease
spot screen is placed between the two sources. For the
grease spot to become indistinguishable from both the
sides, it should be placed at
(a) 80 em from 16 cd lamp and 20 em from 1 cd
(b) 20 em from the 16 cd and 80 em from 1 cd
400 100
(c) - - em from 16 cd and - em from 1 cd
3 3
(d)
100
em from 16 cd and
400
em from 1 cd
3 3
11. A point source of light moves in a straight line parallel
to a plane table. Consider a small portion of the t able
directly below the line of mowment of the source. The
illuminance at this portion varies w1tn this d1stance r from
the source as ~ -
(a)
1
r
1
(b) oc -;z
1 1
(c) oc ~ (d) =-;A
12. As the wavelength is increased from violet to red, the
luminosity
(a) continuously increases
(b) continuously decreases
(c) increases then decreases
(d) decreases then increases
Reflection of Light
13. A dentist has a small mirror of focal length 16 mm. He
views the cavity in the tooth of a patient by holding
the mirror at a distance of 8 mm from the cavity. The
magnification is
(a) 1
(c) 2
(b) 1.5
(d) 3
14. Given width of aperture = 3 mm and 1c = 500 nm. For
what distance ray optics is good approximation?
(a) 18m (b) 18 mm
(c) 18A (d) 18 light years
15. The separation between the screen and a plane mirror
is 2r. An isotopic point source of light is placed exactly
mid way between the mirror and the screen. Assume that
mirror reflects 100% of incident light. Then the ratio of
illuminance on the screen with and without the mirror is
(a) 10 : 1 (b) 2 : 1
(c) 10 : 9 (d) 9 : 1
16. An object is placed asymmetrically between two plane
mirrors inclined at an angle of 72. The number of images
formed is
(a) 5 (b) 4
(c) 2 (d) infinite
17. From a spherical mirror, the graph of 1/v versus 1/u is
1 1
v v
(a) (b)
~
0
Ti
0
u
1
v
F
(c) (d)
0
Ti 0
18. For a convex mirror, the variation of u versus v is given
by
v v
(a) (b)
0
u
0
u
v v
(c)
~
(d)
~
0
u
0
, u
Chapter 23 Ray Optics 907
19. A fish is a little away below the surface of a lake. If the
critical angle is 49, then the fish could see things above
water surface within an angular range of 8 where
Air
Water
e 49
(a) 8=49
"(c) 8=24_!_
0
4
(b) e = 98
(d) 8=90
20. A car is fitted with a convex mirror of focal length 20 em.
A second car 2 m broad and 1.6 m height is 6 em away
from the first car. The position of the second car as seen
in the mirror or the first car is
(a) 19.35 em (b) 17.45 em
(c) 21.48 em (d) 15.49 em
21. A convex mirror forms an image one-fourth the size of the
object. If object is at a distance of 0.5 m from the mirror,
the focal length of mirror is
(a) 0.17 m (b) -1.5 m
(c) 0.4 m (d) - 0.4 m
22. A person 6 feet in length can see his full size erect image
in a mirror 2 feet in height. This mirror has to be
(a) plane or convex (b) plane or concave
(c) necessarily convex (d) necessarily concave
23. A point object is placed at a distance of 30 em from a convex
mirror of a focal length 30 em. The image will form at
24.
25.
(a) infinity
(b) pole
(c) 15 em behind the mirror
(d) no image will be formed
An object is placed at a distance of 10 em from a concave
mirror of radius of curvature 0.6 m. WJ:tich ofthe following
statements is incorrect?
(a) The image is formed at a distance for 15 em from the
mirror.
(b) The image formed is real.
(c) The image is 0.5 times the size of the object.
(d) The image is 1.5 times the size of the object.
With a concave mirror, an object is placed at a distance x
1
from the principal focus, on the principal axis. The image
is formed at a distance x
2
from the principal focus. The
focal length of the mirror is _
(b)
xl +xz
2
(d) Jx
1
x
2
26. A man has a concave shaving mirror of focal length 0.2 m.
How far should the mirror be held from his face in order
to give an image of two fold magnification?
(a) 0.1 m (b) 0.2 m
(c) 0.3 m (d) 0.4 m
908 Chapter 23 Ray Optics
2 7. To focal length of a concave mirror is 12 em. Where should
an object of length 4 em be placed so that an image 1 em
. long is formed?
(a) 48 em
(c) 60 em
(b) 3 em
(d) 15 em
28. The focal length of a concave mirror is 20 em. Where an
object must be placed to form an image magnified two
times when the image is real?
(a) 30 em from the mirror
(b) 10 em from the mirror
(c) 20 em from the mirror
(d) 15 em from the mirror
29. A spherical mirror forms diminished virtual image of
magnification 1/3. Focal length is 18 em. The distance of
the object is
(a) 18 em
(c) 48 em
(b) 36 em
(d) infinite
30. Sun subtends an angle of 0.5 at the centre of curvature
of a concave mirror of radius of curvature 15 m. The
diameter of the image of the sun formed by the mirror is
(a) 8.55 em (b) 7.55 em
(c) 6.55 em (d) 5.55 em
31. A convex mirror and a concave mirror has radii of
curvature 10 em each are placed 15 em apart facing each
other. An object is placed midway between them. If the
reflection first takes place in the concave mirror and then
in convex mirror. the position of the final image is
(a) on the pole of the convex mirror
(b) on the pole of the concave mirror
(c) at a distance of 10 em from convex mirror
(d) at a distance of 5 em from concave mirror
32. An object 5 em tall is placed 1 m from a concave spherical
mirror which has a radius of curvature of 20 em. The size
of the image is
(a) 0.11 em
(c) 0.55 em
(b) 0.50 em
(d) 0.60 em
33. A convex mirror of radius of curvature 1.6 m has an object
placed at a distance of 1 m from it. The image is formed at
a distance of
(a) 8/13 min front of the mirror
(b) 8/13 m behind the mirror
(c) 4/9 min front of the mirror
(d) 4/9 m behind the mirror
34. A short linear object of length b lies along the axis of a
concave mirror of focal length f at a distance u from the
pole of the mirror. The size of the image is equal to
(a) b( u;ffz
(c) b(u;f)
(b) b(_L)1/2
u-f
(d)
35. When an object is kept at a distance of 30 em from a
concave mirror, the image is formed at a distance of
10 em. If the object is moved with a speed of 9 ms-
1
, the
speed with which images moves is
(a) 0.1 ms-
1
(b) 1 ms-
1
(c) 3 ms-
1
(d) 9 ms-
1
,
36. Two plane mirrors are inclined to each other at an angle 8.
A ray of light is reflected first at one mirror and then at
the other. The total deviation of the ray is
(a) 28 (b) 240- 28
(c) 360-28 (d) 180-8
37. A plane mirror is approaching you at 10 cms-
1
. Your image
shall approach you will a speed of
(a) + 10 cms-
1
(b) - 10 cms-
1
(c) + 20 cms-
1
(d) - 20 cms-
1
38. A candle is placed before a thick plane mirror. When
looked obliquely in the mirror, a number of images are
seen from the surfaces of the plane mirror. Then
(a) first image is brightest
(b) second image is brightest
(c) third image is brightest
(d) all images beyond second are brighter
39. An object is approaching a plane mirror at 10 cms-
1
A
stationary observer sees the image. At what speed will the
image approach the stationary observer?
(a) 10 cms-
1
(b) 5 cms-
1
(c) 20 cms-
1
(d) 15 cms-
1
40. A small object is placed 10 em in front of a plane mirror.
If you stand behind the object, 30 em from the mirror and
look at its image, for what distance must you focus your
eyes?
(a) 20 em (b) 60 em
(c) 80 em (d) 40 em
41. When a convergent beam of light is incident on a plane
mirror, the image formed is
(a) upright and real (b) upright and virtual
(c) inverted and virtual (d) inverted and real
42. It is necessary to illuminate the bottom of a well by
reflected solar beam when the light is incident at an
angle of a = 40 to the vertical. At what angle p to the
horizontal should a plane mirror be placed?
(a) 70 (b) 20
(c) 50 (d) 40
43. The sun (diameter d) subtends an angle 8 radian at the
pole of a concave mirror of focal length f . The diameter of
the image of sun formed by mirror is
(a) 8f
(b) f
2
(c) 28 f
(d) f
n
44. A plane mirror reflects a pencil of light to form a real
image. Then the pencil of light incident on the mirror is
(a) parallel (b) convergent
(c) divergent (d) Any of these
45. A spherical mirror forms an image of magnification 3. The
object distance, if focal length of mirror is 24 em, may be
(a) 32 em, 24 em (b) 32 em, 16 em

/
/
/
Refraction of Light
46. How will the image formed by a convex lens be
affected, if the central portion of the lens is wrapped
in blank paper, as shown in the figure .
(a) No image will be formed
(b) Full image will be formed but is less bright
(c) Full image will be formed but without the central
portion
(d) 1\vo images will be formed, one due to each exposed
h a ~
47. The distance v of the real image formed by a convex lens is
measured for various object distance u. A graph is plotted
between v and u . Which one of the following graphs is
correct?
v
v
\
(a) (b)
u
u
v
' ~ (c) (d)
u
u
48. If the space between the lenses in the lens combination
shows were filled with water, what would happen to the
focal length and power of the lens combination?
;
/
/
/ Focal Length
(a) Decreased
I
(b) Decreased
(c) Increased
(d) Increased
Power
increased
unchanged
unchanged
decreased
49. 1\vo convex lenses placed in contact form the image of a
distant object at P. If the lens B is moved to the right, the
image will
A B
(a) move to the left
(b) move to the right
(c) remain ,at P
p
(d) move either to the left or right, depending upon focal
,
length of the lenses
Chapter 23 Ray Optics 909
50. A layered lens as shown in figure is made of two types
of transparent materials indicated by different shades.
A point object is placed on its axis. The object will form
(a) 1 image
(c) 3 images
(b) 2 images
(d) 9 images
51. As shown in figure position of an images I of an object 0
formed by lens. This is possible if
oLJ
(a) a convex lens is placed to the left of 0 .
(b) a concave lens is placed to the left of 0
(c) a convex lens is placed between 0 and I
(d) a concave lens is placed to the right of I
52. The relation between n
1
and n
2
if the behaviour of light
ray is as shown in the figure.
~ e n s
(a) n
2
> n
1
(b) n
1
>> n
2
(c) n
1
> n
2
(d) n
1
= n
2
53. A convex lens A of focal length 20 em and a concave lens
B of focal length 56 em are kept along the same axis with
the distance d between them. If a parallel beam of light
falling on A leaves Bas a parallel beam, tnen distanced in
em will be
(a) 25
(c) 30
(b) 36
(d) 50
54. As shown in figure, the liquids L
1
, L
2
and L
3
have refractive
indices 1.55, 1.50 and 1.20 respectively. Therefore, the
arrangement corresponds to
(a) biconvex lens
(b) biconcave lens
(c) concavo-convex lens
(d) convexo-concave lens
55. A thin double convex lens has radii of curvature each of
magnitude 40 ern and is made of glass with fl = 1.65. The
focal length of the lens in nearly
(a) 30 ern (b) 31 ern
(c) 40 ern (d) 41 ern
910 Chapter 23 Ray Optics
56. A convex lens of focal length f produces a virtual image
n times the size of the object. Then the distance of the
object from the lens is
(a) (n- 1) f
(b) (n + 1) f
(c)
(d) (n:1)f
57. A concave lens of focal length 20 em produces an image
half in size of the real object. The distance of the real
object is
(a) 20 em (b) 30 em
(c) 10 em (d) 60 em
58. An object 15 em high is placed 10 em from the optical
centre of a thin lens. Its image is formed 25 em from the
optical centre on the same side of the lens as the object.
The height of the image is
(a) 2.5 em (b) 0.2 em
(c) 16.7 em (d) 37.5 em
59. One surface of a lens is convex and the other is concave.
If the radii of curvature are r
1
and r
2
respectively, the lens
will be convex, if
(a) r
1
> r
2
(b) r
1
= r
2
(c) r
1
< r
2
(d) r
1
= 1/r
2
60. A lens forms a virtual image 4 em away from it when an
object is placed 10 em away from it. The lens is a .... .lens
offocallength ...
(a) concave, 6.67 em
(b) concave, 2.86 em
(c) convex, 2.86 em
(d) May be concave or convex, 6.67 em
61. A convex lens of focal length I_ m forms a real, inverted
3
image twice in size of the object. The distance of the
object form the lens is
(a) 0.5 m
(c) 0.33 m
(b) 0.166 m
(d) 1m
62. A object is placed at a distance offj2 from a convex lens
of focal length f. The image will be
(a) at one of the foci, virtual and double its size
(b) is greater than 1.5 but less than 2.0
(c) at 2f, virtual and erect
(d) None of the above
63. Consider an equiconvex lens of radius of curvature R
and focal length f. Iff> R, the refractive index I! of the
material of the lens
(a) is greater than zero but less than 1.5
(b) is greater than 1.5 but less than 2.0
(c) is greater than one but less than 1.5
(d) None of the above
64. A convex lens forms an image of an object placed 20 em
away from it at a distance of 20 em on the other side of
the lens. If the object is moved 5 em towards the-lens, the
, image will move
(a) 5 em towards the lens
(b) 5 em away from the lens
(c) 10 em towards the lens
(d) 10 crrf away from the lens
65. A convex lens is placed in contact with a mirror as shown
in figure. If the space between them is filled with water,
its power will
(a) decrease
(b) increase
(c) remain unchanged
(d) increase or decrease depending on the focal length
66. The power of a thin convex lens Cang = 1.5) is + 5.0 D.
When it is placed in a liquid of refractive index ani, then
it behaves as a concave lens of focal length 100 em. The
refractive index of the liquid ani will be
(a) 5/3 (b) 4/3
(c) F3 (d) 5/4
67. A concave lens with unequal radii of curvature made
of glass

= 1.5 ) has a focal length of 40 em. If it is
immersed in a liquid of refractive index 1-ti = 2, then
(a) it behave like a convex lens of 80 em focal length
(b) it behave like a concave lens of 20 em focal length
(c) its focal length becomes 60 em
(d) nothing can be said
68. An object is put at a distance of 5 em from the first focus
of a convex lens of focal length 10 em. If a real image is
formed, its distance from the lens will be
(a) 15 em (b) 20 em
(c) 25 em (d) 30 em
69. A virtual image twice as long as the object is formed by a
convex lens when the object is 10 em away from it. A real
image twice as long as the object will be formed when it
is placed at a distance .... from the lens
(a) 40 em (b) 30 em
(c) 20 em (d) 15 em
70. An achromatic convergent doublet of two lenses in
contact has a power of+ 2 D. The convex lens has power
+5 D. What is the ratio of the dispersive powers of the
convergent and divergent lenses?
(a) 2 : 5 (b) 3 : 5
(c) 5 : 2 (d) 5 : 3
71. If represents refractive index when a light ray goes from
medium ito mediumj, then the product



is equal to
(a) (b)
(c)
1

72. What. is the relation between refractive indices

and


if the behaviour of light rays is as shown in figure.
(a) < =
(c)
(b) < =
(d) > >
73. Monochromatic light of wavelength A.
1
travelling in medium
of refractive index n
1
enters a denser medium of refractive
index n
2
The wavelength in the second medium is
(a) A
1
( :: J (b) A
1
( J
(c) 1.1 (d) A1( J
74. A bucket contains some transparent liquid and its depth is
40 em. On looking from above, the bottom appears to be
raised up by 8 em. The refractive index of the liquid is
(a) 5/4 (b) 5
(c) 4/5 (d) 8/5
75. What is the angle of incidence for an equilateral prism of
refractive index /3 so that the ray is parallel to the base
inside the prism?
(a) 30
(c) 60
(b) 45
(d) Either 30 or 60
76. A lens of refractive index n is put in a liquid of refractive
index n'. If focal length of lens in air isf, its focal length in
liquid will be
(a) fn'(n -1)
n'-n
n'(n-1)
(c) ___:_ _ __:_
j(n'-n)
(b) J(n' -n)
n'(n -1)
fn'n
(d)
n-n'
77. ;;;;
critical angle 8c. If thin layer of l :
water (1-1 = J is now poured on '
the glass air surface, the angle at which the ray emerges
into air at the water-air surface is
(a) 60 (b) 45
(c) 90 (d) 180
78. Light is incident from a medium X at an angle of
incidence i and is refracted into a medium Y at angle of.
refraction r. The graph sin i versus sin r is shown in
figure. Which of the following conclusions would fit
the situation?
1. Speed of light in medium
Y is .J3 times that in
d
0.2
me iumX. sin r
2. Speed of light in medium
Y is 1/ .J3 times that in
medium X.
3. Total internal reflection will
occur above a certain i value.
(a) 2 and 3 (b) 1 and 3
(c) 2 only (d) 3 only
sin i
0.4
79. When a glass slab is placed on a cross made on a sheet,
the cross appears raised by 1 em. The thickness of the
glass is 3 em. The critical angle for glass is
(a) sin-
1
(0.33) (b) sin-:-
1
(0.5)
(c) sin-
1
( .67) (d) sin-
1
(-h/2)
Chapter 23 Ray Optics 911
80. Monochromatic light of frequency 5 x 10
14
Hz travelling
in vacuum enters a medium of refractive index 1.5. It
wavelength in the medium is
(a) 4000 A (b) 5000A
(c) 6000A (d) 5500A
Optical Instruments
81. The magnifying power of a telescope is 9. When it is
adjusted for parallel rays, the distance between the
objective and the eye-piece is found to be 20 em. The
focal lengths of the lenses are
(a) 18 em, 2 em (b) 11 em, 9 em
(c) 10 em, 10 em (d) 15 em, 5 em
82. In compound microscope, magnifying power is 95 and
the distance of object from objective lens is -
1
- em.
1 3.8
The focal length of objective lens is - em. What is the
4
magnification of eye piece?
(a) 5 (b) 10
(c) 100 (d) 200
83. The focal lengths of the objective and eyelenses of a
microscope are 1.6 em and 2.5 em respectively. The
distance between the two lenses is 21.7 em. If the final
image is formed at infinity, the distance between the
object and the objective lens is
(a) 1.8 em (b) 1.70 em
(c) 1.65 em (d) 1.75 em
84. Two points, separated by a distance of 0.1 mm, can just
be inspected on a microscope when light of wavelength
6000 A is used. If the light of wavelength 4800 A is used,
the limit of resolution is
(a) 0.8 mm (b) 0.08 mm
(c) 0.1 mm (d) 0.04 mm
85. The diameter of moon is 3.5 x 10
3
km and its distance
from the earth is 3.8 x 10
5
km. The focal length of the
objective and eye-piece are 4 m and 10 em respectively.
The diameter of the image of the moon will be
approximately
(a) 2 (b) 21
(c) 40 (d) 50
86. With diaphragm of the camera lens set atf/2, the correct
exposure time is 1/ 100 s. Then with diaphragm set atf/8,
the correct exposure time is
(a) l/100 s (b) l / 400 s
(c) 1/200 s (d) 16/100 s
87. An object is viewed through a compound microscope and
appears in focus when it is 5 mm away from the objective
lens. When a sheet of transparent material 3 mm thick
is placed between the objective and the microscope, the
objective lens has to be moved 1 mm to bring the object
back into the focus. The refractive index of the transparent
material is
(a) 1.5
(c) 1.8
(b) 1.6
(d) 2.0
88. A hypermetropic person having near point at a distance of
0.75 m puts on spectacles of power 2.5 D. The near point
now is at
(a) 0.75 m
(c) 0.26 em
(b) 0.83 m
(d) 0.26 m
912 Chapter 23 Ray Optics
89. An astronomical telescope has a converging eye-piece of
focal length 5 ern and objective of focal length 80 em. When
the final image is formed at the least distance of distinct
vision (25 em), the separation between the two lenses is
(a) 75.0 em (b) 80.0 em
(c) 84.2 em (d) 85.0 em
90. The focallengthofobj ective and eye lensof:mastronomical
telescope are respectively 2 m and 5 em. Final image is
formed at (1) least distance of distinct vision (2) infinity.
Magnifying powers in two cases will be
(a) - 48,-40 (b) - 40, 48
(c) -40, + 48 (d) -48, + 40
91. A man' s near point is 0.5 m and far point is 3 m. Power
spectacle lanses repaired for
(i) reading purposes
(ii) seeing distant objects, respectively.
(a) -2 D and + 3D (b) + 2 D and- 3 D
(c) + 2 D and- 0.33 D (d) -2 D and + 0.33 D
92. A hypermetropic person has to use a lens of power + 5 D to
normalise his vision. The near point of the hypermetropic
eye is
(a) 1m
(c) 0.5 m
(b) 1.5 m
(d) 0.66 m
93. A compound microscope has an obj ective and eye-piece
as thin lenses of focal lengths 1 em and 5 em respectively.
The distance between the objective and the eye-piece is
20 em. The distance at which the object must be placed
infront of the objective if the final image is located
at 25 em from the eye-piece, is numerically
(u) 95/6 em ' (b) 5 em
(c) 95/ 89 em (d) 25/6 em
94. The focal length of the objective and the eye-piece of a
microscope are 4 mm and 25 mm respectively. If the final
image is formed at infinity and the length of the tube is
16 em, then t he magnifying power of microscope will be
(a) -337.5 (b) -3.75
(c) 3.375 (d) 33. 75
95. A simple microscope consists of a concave lens of power
- 1 OD and a convex lens of power + 20D in contact. If the
image is formed at infinity , then the magnifying power
CD= 25 em is
(a) 2.5
(c) 2.0
(b) 3.5
(d) 3.0
96. The magnifying power of an astronomical telescope is 10
and the focal length of its eye-piece is 20 em. The focal
length of its objective will be
(a) 200 em (b) 2 em
(c) 0.5 em (d) 0.5 x 10-
2
em
Prism
97. For a prism, its refractive index is cos A. Then minimum
2
angle of deviation is
(a) 180 -A (b) 180- 2A
(c) 90 - A (d) A
2
98. The refractive index of a prism for a monochromatic
wave is J2 and its refracting angle is 60. For minimum
deviation, the angle of incidence will be
99.
100.
101.
(a) 30
(c) 60
It is desired to make a converging achromatic combination
of mean focal length 50 em by using two lenses of
materials A and B. If the dispersive powers of A and B
are in ratio 1: 2, the focal lengths of the convex ar.d the
concave lenses are respectively
(a) 25 em and 50 em (b) 50 em and 25 em
(c) 50 em and 100 em (d) 100 em and 50 em
Two parallel light rays are incident at one surface of a
prism of refractive index 1.5 as shown in figure. The
angle between the emergent rays is nearly
(a) 19 (b) 37
(c) 45 (d) 49
The refractive index of the material of a prism is J2 and
the angle of prism is 30. One of its refracting faces is
polished. The incident beam of light will retrace back for
angle of incidence
(a) 0
(c) 60
(b) 45
(d) 90
102. The cross-section of a glass prism has the form of an
isoceles triangle. One of the refracting faces is silvered.
A ray of light falls normally on the other refracting face .
After being reflected twice, it emerges through the base of
the prism perpendicular to it. The angles of the prism are
(a) 54, 54, no (b) no, no, 36
(c) 45, 45, 90 (d) 57, 57, 76
103. Parallel beam containing light of'), = 400 nm and 500 nm
is incident on a prism as shown in figure. The refractive
index fl of the prism is given by the relation
1-!C"-) = 1.20 + o.s x ~ o - 1 4
I.
Which of the following statement
is correct?
(a) Light on= 400 nm undergoes
total internal reflection.
(b) Light of A. = 500 nm undergoes
total internal reflection.
10
,'
(c) Neither of the two wavelengths undergoes total
internal reflection.
(d) Both wavelengths undergoes total internal reflection.
104. The maximum refractive index of a prism which permits
the passage of light through it, when the refracting angle
of the prism is 90, is
(a) J3
(c) J3
2
(b) J2
(d) ~
105. An object is placed 30 em to the left of a diverging lens
whose focal length is of magnitude 20 em. Which one of
the following correctly states the nature and position of
the virtual image formed?
Nature of image
(a) inverted, enlarged
(b) erect, diminished
(c) inverted, enlarged
(d) erect, diminished
(e) inverted, enlarged
Distance from lens
60 em to the right
12 em to the left
60 em to the left
12 em to the right
12 em to the left
Chapter 23 Ray Optics 913
Exercise II
Only One Correct Option
1. A ray oflight passes through four transparent medium with
refractive indices J.1
1
, J.1
2
, J.1
3
and J.1
4
as shown in the figure.
The surfaces of all media are parallel. If the emergent ray
CD is parallel to the incident ray AB. we must have
1!3
(a) J.11 == J.12 (b) J.12 == J.l3
(c) J.13 == J.l4 (d) J.13 == J.11
2. A diminished image of an object is to be obtained on
a screen 1.0 m away from it. This can be achieved by
approximately placing
(a) a convex mirror of suitable focal length
(b) a concave mirror of suitable focal length
(c) a convex lens of focal length less than 0.25 m
(d) a concave lens of suitable focal length
3. A ray of light passes through an equilateral prism such
that the angle of incidence and the angle of emergence
are both equal to 3/ 4 th of the angle of prism. The angle
of minimum deviation is
(a) 15 (b) 30
(c) 45 (d) 60
4. A lens of focallengthf projects i'rt:tiines magnified image
of an object on.a scnen. the distance of the screen from
the lens is
(a) 1) (b) (m: 1)
(c) f (m -1) (d) f (m + 1)
5. A thin equiconvex lens of refractive index 3/ 2 and radius
of curvature 30m is put in water (refractive index =
Its focal length is
(a) 0.15 m (b) 0.30 m
(c) 0.45 m (d) 1.20 m
6. A 16 em long image of an object is formed by a convex
lens on a screen. On moving the lens towards the screen,
without changing the positions of the object and the
screen, a 9 em long image is formed again on the screen.
The size of the object is
(a) 9 em (b) 11 em
(c) 12 em (d) 13 em
7. TWo lenses, one concave and the other convex of same
power' a:re placed such that their principal axes coincide.
If the separation between the lenses is x, then
(a) real image is formed for X== o<&tily
(b) real image is formed for all vafues of x
(c) system will behave like a for x = 0
(d) virtual image is formed for a'liSV.alues of x other than
. .Jr.
zero '
8. A ray of light falls on a transparent glass with
refractive index (relative to air) of 1.62. The angle of
incidence for which the reflected and refracted rays are
mutually perpendicular is
(a) tan-
1
(1.62)
(c) cos-
1
(1.62)
(b) sin-
1
(1.62)
(d) None of these
9. A double convex lens made out of glass (refractive index
J.1 == 1.5) has both radii of curvature of magnitudes 20 em.
Incident light rays parallel to the axis of this lens will
converge at a distance d such that
20
(a) d = 10 em (b) d =-em
3
(c) d == 40 em (d) d == 20 em
10. A 4 em thick layer of ,--,...,---,-----
water covers a 6 em
thick glass slab. A coin
is placed at the bottom
of the slab and is being
observed from the air
side along the normal
to the surface. Find _the
4cm
apparent position of the coin from
(a) 7.0 em (b) 8.0 em
(c) 10 em (d) 5 em
Coin
11. The light takes in travelling a distance of 500 m in
water. Given that J.1 for water is 4/ 3 and the velocity of
light in vacuum is 3 x 10
10
cms-
1
Calculate equivalent
optical path.
(a) 566.64 m (b) 666.64 m
(c) 586.45 m (d) 576.64 m p
12. How many images are formed by the lens shown, if an
object is kept on its axis?
(a) 1
(c) 3
1!1
(b) 2
(d) 4
13. For a optical arrangement shown in the figure. Find the
position and nature of image.
(a) 32 em
(c) 6 em
(b) 0.6 em
(d) 0.5 em
u
14. A thin plano-convex lens of focal length f is split into two
halves. One of the halves is shifted along the optical axis.
The separation between object and image plane is 1.8 m.
The magnification of the image fonned by one Of the half
lens is 2. Find the focal-length of the lens and separation
between the two halves.
I :
I
914 Chapter 23 Ray Optics
(a) 0.1 m
(c) 0.9 m
(b) 0.4 m
(d) 1m
15. The refractive indices of the crown glass for blue and red
light are 1.51 and 1.49 respectively and those of the flint
glass area 1. 77 and 1. 73 respectively. An isosceles prism
of angle 6 is made of crown glass. A beam of white light
is incident at a small angle on this prism. The other flint
glass isosceles prism is combined with the crown glass
prism such that there is no deviation of the incident
light. (i) Determine the angle of the flint glass prism.
(ii) Calculate the net dispersion of the combined system.
(a) -4, 0.04, (b) 4, 0.04
(c) 5, 0.04 (d) -5, 0.04
16. A plano-convex lens has a thickness of 4 em. When placed
on a horizontal table, with the curved surface in contact
with it, the apparent depth of the bottom most point of the
lens is found to be 3 em. If the lens is inverted such that the
plane face is in contact with the table, the apparent depth of
uJ L the centre of the plane face is found to be 25/ 8 em. Find the
focal length of the lens. Assume thickness to be negligible
(a) 85 em (b) 59 em
(c) 75 em (d) 7.5 em
17. A convex lens of focallengthf is placed some where in
between an object and a streen. The distance between
object and screen is x. If numerical value of magnification
produced by lens is m, focal length of lens is
to w,
2r br(a)
mx
(b)
(m-1)
2
mx
(m+1f
(m+ 1)
2
(c) x
(m-1f
(d) X
m m
18. A ray oflight from a denser medium strikes a rarer medium
at angle of incidence i. The reflected and refracted rays
make an angle of 90 with each other. The angles of
2
j f::.J:. !eflection and refraction are r and r' respectively. The
critical angle is
(a) sin-
1
(tanr') (b) sin-
1
(tanr)
(c) tan-
1
(tanr')
19. If eye is kept at a depth h inside water of refractive
21 2
[{ : d d . d "d th h d" f h . l
1
, m ex an VIewe outs1 e, en t e 1ameter o t e crrc e
,r ' through which the outer objects become visible, will be
(a) _h_ (b) _h_

ri!gn9J'. . 2 h h
suld 'c) 2 (d) G
-1 vw
20. A ray of light is incident at 60 on one face of a prism
whi.ch has angle 30. The angle between the emergent ray
and incident ray is 30. What is the angle between the ray
and the face from. which its emerges?
(a) 0 (b) 30
(c) 60 (d) 90
21. An object is kept at a distance of 16 em from a thin lens
and the image formed is real. If the object is kept at a
distance of 6 em from the same lens, the image formed
is virtual. If the sizes of the images formed are equal the
focal length of the lens will be
(a) 21 em (b) 11 em
(c) 15 em (d) 17 em
22. P is a point on the axis of a concave mirror. The image of
P formed by the mirror, coincides with P. A rectangular
glass slab of thickness t and refractive index f.l is now
introduced between P and the mirror. For the image of P
to coincide with P again, the mirror must be moved
(a) towardsPby
(b) awayfromPby
(c) towardsPby J
(d) awayfromPby J
23. A point object is placed at a distance of '25 em from
a convex lens of focal length 20 em. If a glass slab of
thickness t and refractive index 1.5 is inserted between
the lens and the object, the image is formed at infinity.
The thickness t is
(a) 15 em
(c) 10 em
(b) 5 em
(d) 20 em
24. A plano convex lens fits exactly into a plano concave lens.
Their plane surfaces are parallel to each other. If the lenses
are made of different materials of refractive indices f1
1
and
f.l
2
and R is the radius of curvature of the curved surface of
the lenses, then focal length of the combination is
(a) R (b) R
+
R 2R
(c) (d)
-
25. One of the refracting surfaces of a prism of angle 30 is
silvered. A ray of light incident at an angle of 60 retraces
its path. The refractive index of the material of prism is
(a) (b) 3/2
(c) 2 (d) ifi
26. The focal length of objective and eye-piece of a microscope
are 1 em and 5 em respectively. If the magnifying power
for relaxed eye is 45, then length of the tube is
(a) 9 em (b) 15 em
(c) 12 em (d) 6 em
27. A glass prism ABC (refractive index 1.5) , immersed in
water (refractive index 4/3) . A ray of light is incident
normally on face AB. If it is totally reflected at face AC
then
-:_-------. c _-_-_-_-_-_-_-_-_-_-.:: .::_-_-_-.
(a)
. e 8
(b)
. e 2
Sill 2':- Sill 2':-
9 3
sine= F3
(d)
2 .
9
8
(c)
- <Sill < -
2
3 9
28. A plane mirror is placed at the bottom of a tank containing
a liquid of refractive index fl P is a small object at a
height h above the mirror. An observer 0 -vertically above
P outside the liquid sees P and its image in a mirror. The
apparent distance between these two will be
29.
(a) 2 fl h
(c) _3!!_
J.L-1
o.
I
I
I
I
I
Light takes t
1
second to travel a distance x in vacuum and
the same light takes t
2
second totravel10xcminamedium.
Critical angle for corresponding medium will be
(a) sin-1 ( J
(c) sm -- sm -- . -1(10t
1
J (d) . -1( t1 J
t2 10t
2
30. A plano convex lens of (f = 20 em) is silvered at plane
surface. New fwill be
(a) 20 em
(c) 30 em
(b) 40 em
(d) 10 em
May have More than One Correct Option
A point object is at 30 em from a convex glass lens
(lls =%) of focal length 20 em. The final image of object
will be formed at infinity if
(a) another concave lens of focal length 60 em is placed
in contact with the previous leJ:lS
(b) another convex lens of focal length 60 em is placed at
a distance of 30 em from the first lens
(c) the whole system is immersed in a liquid of refractive
index 4/3 ,.
(d) the.whole system is immersed i:p. fi liquid of refractive
index 9;8 ... u.
32. For which of the pairs of u and f for a mirror image is
smaller in size.
(a) u = - 10 em, f = 20 em
(b) u = - 20 em, f = - 30 em
(c) u =- 45 em, f = -10 em
(d) u =- 60 em, f = 30 em
33. There are three optical media, 1, 2 and 3 with their refractive
indices fll j> f.!z > f.!
3
. (TIR- total internal reflection)
Chapter 23 Ray Optics 915
(a) When a ray of light travels from 3 to 1 no TIR will
take place \
(b) Critical angle between 1 and 2 is less than the critical
angle between 1 and 3
(c) Critical angle between 1 and 2 is more than the critical
angle between 1 and 3
(d) Chances ofTIR are more when ray oflight travels from
1 to 3 compare to the case when it travel from 1 to 2
34. Parallel rays oflight are falling on convex spherical surface
of radius of curvature R = 20 em as shown. Refractive
index of the medium is fl = 1.5. After refraction from the
spherical surface parallel rays

(a) actually meet at some point
(b) appears to meet after extending the refracted rays
backwards
(c) meet (or appears to meet) at a distance of 30 em from
the spherical surface
(d) meet (or appears to meet) at a distance of 60 em from
the spherical surface
35. A ray of light travelling in a transparent medium falls on
a surface separating the medium from air, at an angle of
incidence of 45. The ray undergoes total internal reflection.
If n is the refractive index of the medium with respect to
air, select the possible values of n from the following
(a) 1.3 (b) 1.4
(c) 1.5 (d) 1.6
.'\f
Comprehension Based Questions
Passage I
I
The power of a convex lens depends on the radius of
curvature and refractive index of lens material a'nd is
given by
Pa = Ca llg -1) (

-

} when lens is in air.
The refractive index of material is roughly.given by JH
B
Jl =A+ /.} (Cauchy's formula).
1
If a lens is dipped in a liquid, its power is changed and is
given by
= C;J.tg -1)( - J:;
36. A lens (f.!g=l.5) in air has a power + 2 D. When
dipped in water of refractive index 4/3, its will
become/remain
1
(a) + 4 D (b) + 8 D "'
(c)+ 2D (d) -4D
3 7. The refractive index of glass for yellow light of wavelength
6000 A is 1.5. Then the refractive index of glass fo1 blue
light A = 4000 A will become/remain
(a) 1.5 (b) less than 1.5
(c) greater than 1.5 (d) information is insufficient
38. If a hollow convex shaped glass is filled with water and
surrounding is glass. The lens will act as
(a) convex lens (b) concave lens
(c) glass plate (d) convex mirror
\
916 Chapter 23 Ray Optics
Passage II
Total internal refl'ection is the phenomenon of reflection
of light into denser medium at the interface of denser
medium with a rarer medium. Light must travel from
denser to rarer and angle of incidence in denser medium
must be greater than critical angle (C) for the pair of
media in contact. We can show that
1
11
=sine
39. Critical angle for water air interface is 48.6. What is the
refractive index of water?
(a) 1 (b) 3/2
(c) 4/3 (d) 3/ 4
40. Light is travelling from air to water at Li = 50, which
is greater than critical angle for air water interface. What
fraction of light will be totally reflected?
(a) 100% (b) 50%
(c) None (d) Cannot say
41. Critical angle for glass air interface where J.! of glass is 3/ 2 is
(a) 41.8 (b) 60
(c) 30 (d) 44.3
42. Critical angle for air water interface for violet colour is
49. Its value for red colour would be
(a) 49 (b) 50
(c) 48 (d) cannot say
43. A point source of light is held at a depth h below the
surface of water. If Cis critical angle of air water interface,
the diameter of circle of light coming from water surface
would be
(a) 2 h tan C
(c) h sin C
Assertion and Reason
(b) h tan C
(d) h/ sin C
Directions Question No. 44 to 50 are Assertion-Reason
type. Each of these contains two Statements : Statement I
(Assertion), Statement II (Reason). Each of these questions
also has. four alternative choice, only one of which is correct.
You have to select the correct choices from the codes (a) , (b),
(c) and (d) given below:
(a) If both Assertion and Reason are true and the Reason is
correct explanation of the Assertion.
(b) If both Assertion and Reason are true but Reason is not
correct explanation of the Assertion.
(c) If Assertion is true but Reason is false .
(d) If Assertion is false but the Reason is true.
0
44. Assertion Convergent lens property of converging remain
same in all mediums.
Reason Property of lens whether the ray is diverging or
converging depends on the surrounding medium.
45. Assertion A concave mirror and convex lens both have
' the same focal length in air. When they are submerged in
water, they will still have the same focal length.
Reason The refractive index of water is greater than the
refractive index of air.
46. Assertion The focal length of the objective of the telescope
is larger than that of eye-piece
Reason The resolving power of telescope increases when
the aperture of objective is small.

47. Assertion A short sighted person cannot see objects
clearly when placed beyond 50 em. He should use a
concave lens of power 2 D.
Reason Concave lens should form image of an object at
infinity placed at a distance of 50 em.
48. Assertion By roughening the surface of a glass sheet its
transparency can be reduced.
Reason Glass sheet with rough surface absorbs more
light.
49. Assertion The refractive index of diamond is /6 and
that of liquid is .J3 . If the light travels from diamond
to the liquid, it will initially reflected when the angle of
incidence is 30.
Reason J.! = - .-
1
- where J.! is the refrective index of
smC
diamond with respect to liquid.
50. Assertion The colour of the green flower seen through
red glass appears to be dark.
Reason Red glass transmits only red light.
Previous Year's questions
51. The distance between an object and a divergent lens ism
times the focal length of the lens. The linear magnification
52.
produced by the lens is (WB JEE 2009)
(a) m (b) 1/m
1
(d)
m+1
(c) (m + 1)
In an optics experiment, with the position of the object
fixed, a student varies the position of a convex lens and
for each position, the screen is adjusted to get a clear
image of the object. A graph between the object distance
u and the image distance v, from the lens, is plotted using
the same scale for the two axes. A straight line passing
through origin and making an angle of 45 with the x-axis
meets the experimental curve at P. The of P
will be (AIEEE 2009)
(a) ( f.f) (b) (j,j)
(c) (4j, 4f) (d) (2/, 2/)
53. A thin lens of ( J.! 1.5) of focal length + 10 em is
immersed in water (J.! = 1.33). The new focal length is
(DCE 2009)
(a) 20 em
(c) 48 em
(b) 40 em
(d) 12 em
54. A telescope consists of two thin lenses of focal lengths
0.3 m and 3 em respectively. It is focused on moon which
subtends an angle of 0.5 at the objective. Then, the angle
subtended at the eye by the final image will be
(a) 5
(c) 0.5
(b) .0.25
(d) 0.35
(UP SEE 2009)
55. A ray oflight passes through an equilateral prism such that
the angle of incidence is equal to the angle of emergence
and the latter is equal to the angle of prism. The angle
of deviation is
4
(UP SEE 2009)
(a) 25 (bJ--:50
(c) 45 (d) 35
56. Two plane mirrors are inclined at an angle e. It is found
that a ray incident on one mirror at any angle is rendered
parallel to itself after reflection from both the mirrors.
The value of 8 is (UP SEE 2009)
(a) 30 (b) 60
(c) 90 (d) 120
57. When a ray of light enters a glass slab from air
(UP SEE 2009}
(a) its wavelength decreases
(b) its wavelength increases
(c) its frequency increases
(d) Neither its wavelength nor its frequency changes
58. Critical angle of light passing from glass to water is
minimum for (UP SEE 2009)
(a) red colour (b) green colour
(c) yellow colour (d) violet colour
59. In a spectrometer experiment, "as wavelength of a spectral
line increases, deviation also increases." This is true
(Karnataka CET 2008)
(a) in grating spectrum
(b) in prism spectrum
(c) Both in grating as well as prism spectrum
(d) Neither in prism spectrum nor in grating spectrum
60. The focal length of a biconvex lens of refractive index 1.5
is 0.06 m. Radius of curvature are in the ratio 1 : 2. Then
radii of curvatures of two lens surface are
(a) 0.045m, 0.09 m
(c) 0.04m, 0.08 m
(Karnataka CET 2006}
(b) 0.09 m, 0.18 m
(d) 0.06m, 0.12 m
61. A person inside water llw =' 4/3 sees the setting sun at
about (Karnataka CET 2008)
(a) 42 to the horizontal
(b) 48 to the horizontal
(c) 42 to the vertical
(d) 24 to the vertical
62. A ray of light is incident on a glass slab of thickness t, at
an angle i, r is the angle of refraction in the glass plate.
Distance travelled in the glass plate is
(a) t cos r
(c) t/cos r
(Karnataka CET 2008)
(b) ttanr
(d) t/sin r
63. A right angled hollow prism is filled with a liquid. A ray
of light entering the prism grazing one face emerges out
grazing other face. Refractive index of the liquid is
(a) 1. 73
(c) 1.54
(b) 1.33
(d) 1.41
(Karnataka CET 2008)
64. The radius of curvature of the convex face of a
plano-convex lens is 15 em and the refractive index of the
material is 1.4. Then the power of the lens in diopter is
(Kerala CET 2008)
(a) 1.6
(c) 2.6
(b) 1.66
(d) 2.66
65. The wavelength of red light from He-Ne laser is 633 nm in
air but 474 nm in the aqueous humor inside the eye ball.
The speed of red light through the aqueous humor is
(Kerala CET 2008)
(a) 3 x 10
8
ms-
1
(b) .1.34 x 10
8
ms-
1
(c) 2.25 x 10
8
ms-
1
(d) 2.5 x 10
8
ms-
1
Chapter 23 Ray Optics 917
66. If the angle of minimum deviation is 60 for an equilateral
prism, then the refractive index of the material of the
prism is (Kerala CET 2008)
(a) 1.41 (b) 1.5
(c) 1.6 (d) 1.33
67. A student measures the focal length of a convex lens by
putting an object pin at a distance u from the lens and
measuring the distance v of the image pin. The graph
between u and v plotted by the student should look like
(AIEEE 2008)
)
v(cm) v(cm)
t
I
t
(a) (b)
"
0
-
0
- u(cm) u(cm)
(c)
' I T ~
(d)
) tm)
0
-
0
- u(cm) u(cm)
68. An experiment is performed to find the refractive index
of glass using a travelling microscope. In this experiment
distances are measured by (AIEEE 2008)
(a) a screw gauge provided on the microscope
(b) a vernier scale provided on the microscope
(c) a standard laboratory scale
(d) a metre scale provided on the microscope
69. Given allg = 3/2 and allw = 4/3. There is an equiconvex
lens with radius of each surface equal to 20 em. There is
air in the object space and water in the image space. The
focal length of lens is (DCE 2008)
(a) 80 em (b) 40 em
(c) 20cm (d) 10 em
70. An observer looks at a tree of height 15m with a telescope
of magnifying power 10. To him the tree appears
(BVP Engg. 2008)
(a) 10 times taller (b) 15 times taller
(c) 10 times nearer (d) 15 times nearer
71. The resolution limit of the eye is 1 min. At a distance x km
from the eye, two persons stand with a lateral separation
of 3m. For the two persons to be just resolved by the
naked eye, x should be (BVP Engg. 2008)
(a) 10 km (b) 15 km
(c) 20 km (d) 30 km
72. A thin prism P
1
with angle 4 and made from glass of
refractive index 1.54 is combined with another thin prism
P
2
made from glass of refractive index 1.72 tb produce
dispersion without deviation. The angle of the prism P
2
is
(a) 5.33
(c) 3
(BVP Engg. 2008)
73. The image formed by a concave mirror (UP SEE 2008)
(a) is always real
(b) is always virtual
(c) is certainly real if the object is virtual
(d) is certainly virtual
I I
.9t8 Chapter 23 Ray Optics
~ ~ ~ A lens made of glass whose index of refraction is 1.60 has
a focal length of + 20 em in air. Its focal length in water,
', whose refractive index is 1.33, will be (UP SEE 2008)
(a) three times longer than in air
(b) two times longer than in air
(c) same as in air
(d) None of the above
( ~ 5 . Magnification at least distance of distinct vision of a
(B) simple microscope having its focal length 5 em is
( ::>) (Kerala CET 2007)
(L) (a) 2 (b) 4
(d) (c) 5 (d) 6
(B) (e) 7
(' .. ,
76.
lL
(B)
(d)
The limit of resolution of an optical instrument arises on
account of (Gujarat CET 2007)
(a) reflection (b) diffraction
(c) polarization (d) interference
77. The length of deviation for a glass prism is equal to its
refracting angle. The refractive index of glass is 1.5. Then
the angle of prism is (J & K CET 2007)
(E) (a) 2 cos-
1
(3/ 4) (b) sin-
1
(3/ 4)
li') (c) 2 sin-
1
(3/ 2) (d) cos-
1
(3/2)
9(. ~ . A convex lens is placed between object and a screen.
::>J 'The size of object is 3 em and an image of height 9 em is
'3
1
obtained on the screen. When the lens is displaced to a new
lB) position, what will be the size of image on the screen?
(Orissa JEE 2007)
(a) 2 em
(c) 4 em
(b) 6 em
(d) 1 em
79. A glass slab (J.! = 1.5) of thickness 6 em is placed over a
paper. What is the shift in the letters? (DCE 2007)
(a) 4 em (b) 2 em
cc) 1 em (d) None of these
80. In a laboratory four convex lenses L
1
, L
2
, L
3
and L
4
of
focal lengths 2, 4, 6 and 8 em, respectively are available.
Two of these lenses form a telescope of length 10 em and
magnifying power 4. The objective and eye lenses are
respectively (UP SEE 2007)
(a) L
2
, L
3
(b) L
1
, L
4
(c) L
1
, L
2
(d) L
4
, L
1
,
------
81. A symmetric double convex lens is cut in two equal parts
by a plane perpendicular to the principle axis. If the
power of the original lens is 4 D, the power of a cut lens
will be (UP SEE 2007)
(a) 2 D (b)- 3D
(c) 4D (d) 5D
82. Focal length of objective and eye piece of telescope are
200 em and 4 em respectively. What is the length of
telescope for normal adjustment? (DCE 2006)
(a) 196 em (b) 204 em
(c) 250 em (d) 225 em
83. The refractive index of glass is 1.520 for .Jed light and
1.525 for blue light. Let D
1
and D
2
be the angles of
minimum deviation for red light and blue light respectively
in a prism of this glass. Then {A IEEE 2006)
(a) D
1
> D
2
(b) D
1
= D
2
(c) D
1
= D
2
(d) D
1
can be less than or greater than 9-epending upon
the angle of the prism
84. A point object is placed at the centre of a glass sphere of
radius 6 em and refractive index 1.5. The distance of the
virtual image from the surface of sphere is (BHU 2006)
(a) 2 em (b) 4 em
(c) 6 em (d) 12 em
85. The minimum diameter of the cross-section of cone of
light with surface of water due to luminous point 8 ern
below the surface is (J.! = 1.33) (BVP Engg. 2006)
(a) 18.24 em (b) 20.24 em
(c) 25 em (d) 30 em
86. Light of certain colour has 200 waves to the millimeter in
air. What will be the wavelength of this light in medium
of refractive index 1.25? (BVP Eng g. 2006)
(a) 1ooo A Cb) 2ooo A
Cc) 3ooo A Cd) 4ooo A
87. A light moves from denser to rarer medium. Which of the
following is correct? (UP SEE 2006)
(a) Energy increases (b) Frequency increases
(c) Phase changes by 90 (d) Velocity increases
Chapter 23 Ray Optics I
Answers
Exercise I
1. (a) 2. (c) 3. (b) 4. (c) 5. (b) 6. (d) 7. (a) 8. (c) 9. (b) 10.

11. (c) 12. (c) 13. (c) 14. (a) 15. (c) 16. (a) 17. (b) 18. (c) 19. (b) 20. (a)
2;1. (a) 22. (c) 23. (c) 24. (a) 25. (d) 26. (a) 27. (c) 28. (a) 29. (b) 30. (c)
31. (a) 32. (c) 33. (d) 34. (d) 35. (b) 36. (c) 37. (c) 38. (b) 39. (a) 40. (d)
41. (a) 42. (a) 43. (a) 44. (b) 45. (b) 46. (b) 47. (d) 48. (d) 49. (b) 50. (b)
51. (d) 52. (a) 53. (b) 54. (c) 55. (b) 56. (c) 57. (a) 58. (d) 59. (c) 60. (a)
61. (a) 62. (a) 63. (c) 64. (d) 65. (a) 66. (a) 67. (a) 68. (d) 69. (b) 70. \h)
71. (c) 72. (a) 73. (a) 74. (a) 75. (c) 76. (a) 77. (c) 78. (c) 79. (c) 80. (a)
81. (a) 82. (a) 83. (d) 84. (b) 85. (b) 86. (d) 87. (a) 88. (d) 89. (c) 90. (a)
91. (c) 92. (a) 93. (c) 94. (a) 95. (a) 96. (a) 97. (b) 98. (b) 99. (a) 100. (b)
101. (b) 102. (b) 103. (a) 104. (b) 105. (b)
\
Exercise II
1. (a) 2. (c) 3. (b) 4. (d) 5. (d) 6. (c) 7. 8. (a) 9. (d)
. 10. (a)
11. (b) 12. (a) 13. (b) 14. (b) 15. (a) 16. (c) 17. (a) 18. (b) 19. (c) 20. (d)
21. (b) 22. (c) 23. (a) 24.
(c) '
25. (b) 26. (b) 27. (a) 28. (b) 29. (c)
30.
31. (a,d) 32. (a,c,d) 33. (a,c,d) 34. (a,d) 35. (c,d) 36. (b) 37. (c) 38. (b) 39. (c) 40. c)
41. (a) 42. (c) 43. (a) 44. (d) 45. (d) 46. (d) 47. (a) 48. (c) 49. (a) 50. (a)
51. (d) 52. (d) 53. (b) 54. (a) 55. (b) 56. (c) 57. (a) 58. (d) 59. (d) 60. (a)
61. (a) 62. (c) 63. (d) 64. (d) 65. '(c) 66. (e) 67. (d) 68. (b) 69. (b) 70. (c)
71. (a) 72. (c) 73. (c) 74. (a) 75. (d) 76. (b) 77. (a) 78. (d) 79. (b) 80. (d)
81. (a) 82. (b) 83. (a) 84. (c) 85. (a) 86. (d) 87. (d)
,
920 _ 23 Ray Optics
Hints & Solutions
til{ l
w 9't.
p- Exercise I
e
3. o r2 (4)2 16
r h
E = Icos8 = I x (415)
p r'
2
C5i
41
125
E
0
I 125 125
:. -=-X-=-
Ep 16 4I 64
4. At minimwn distance, incidence is normal. Therefore,
I 250
E=-z=-
2
=6.94 lux.
r 6
5. 5x 1o-
4
Icos60
200x200
or
I= 5x 1o-
4
x 4 x 10
4
x 2 = 40
cd
6.
r 40 x 40 16
-=---=-
I 50 x 50 25
1-E.=1- 16 =_2_
I 25 25
I-I' 9
or --x 100=- x 100=36o/o
I 25
7. When the screen is equally illuminated,
E
1
= E
2
or
!J_ = .!1_ or !J_ = r/ = 30 x 30 = _2_
r
1
2
r] I
2
ri 50 x SO 25
1 16
8. 502 =d2
or d
2
= (50i x 16
or d=50 x 4cm=200cm=2m.
9. r
1
= 10 em, r
2
= 8 em
I
1
_ 64
1
I
1
_
1
64
I
2
- 100 ' - I
2
- - 100
or I 2 -I1 = 36
or
I
2
o 100
Iz-IJ x 100'=36o/o
I2
1 16
10. x z (IOO-x)2
or
1 4
x 100-x
or
Sx =-100 or X= 20 em ,
1
or E cx: -
r 3
12. The variation of relative luminosity with wavelength is
shown here.
1.0
Relative
luminosity
400 700
Wavelength (in nm) -
Note that
360
is odd and object line asymmetrically.
.8
1 1 1 1 1 1
17. -+-=-or-=--+-
u v f v u f
Now, compare withy = mx + c
Therefore graph is a straight line having negative slope.
18. Think in terms of rectangular hyperbola.
19. The angular range is clearly twice the critical angle.
1 1 1 1 31
20. -+--=-or-=-
v -600 20 v 600
or
600
v =-em= 19.35
31
21. _!_=_!_
f-u 4
f
f -(-0.5)
or 4f=f+0.5 or 3/=0.5
or
f
0.5
=-m=0.17m
3
22. The image is erect and diminished. So, the mirror in
necessarily convex.
1 1 1
23. -+-=-
-30 v 30
or
1 2 1
v 30 15
or v = 15 em
24. f =-
0

6
= -0.3m = -30cm
2
25.
1 1 1
-+-=-
v -10 -30
1 1 1 3-1
v 10 30 30
30
or v =- em= 15 em
2
v 15
m = -- = --= 1.5
u -10
Object lies between principal focus and pole. So, the
image is virtual and erect.
1 1 1
-+-=-
u v f
1 1 1
--+--=-
f-xl f-x2 f
or
f-x
2
+ f-x
1
_ 1
Cf-xl)(f-x2)
or
f
2
- fx2- fx1 + X1X2 = 2/
2
- f(xl + X2)
or f
2
= x
1
x
2
or f = Jx
1
x
2
This is Newton's mirror formula.
- f
26. m---
or
or
or
f-u
27 m
=_f___
.
f-u
or
28. m =_f___
f-u
,
2 = -0.2
-0.2-u
0.2
2=-- or 0.4+2u=0.2
0.2+u
2u = 0.2-0.4 = -0.2
u = -O.lm
JlW c,,
1 -12 ._n ;l;
--=---=---
4 -12-u 12rou
12 + u = -48 or u = _:66\:m.
- 20
-2=---
-20-u
or
or
Chapter 23 Ray Optics 921

20+u
20+u = -10
u =-30cm.
29. Clearly, the given mirror is a convex mirror.
m =_f___
f-u
1 18
-=--
3 18-u
or 3x18 = 18-u
or u=-2x 18 cmoru =-36crn.
1 1t
30. 8=-x-rad
2 180
diameter of image =
8
focal length
or diameter of image
1 1t 15
=-x-x-x100cm=6 55cm
2 180 2 . .
31. For concave mirror
15
u=--cm v="'
2 ' .
10
f =--em= -Scm
2
1 1 1 1 1
=---=-----
v f u -5 -15/2.
1 2 -1
=--+-=-
5 15 15
or v = -15 em
Clearly; the position of the final image is on the pole of
the convex mirror.
32. f = -10 ern, 0 = 5 em,
u = - 100 em, I = ?
!__ =_f___
0 f-u
-10 -10
I= xs =-xs ern
-10-(-100) 90
= -0.55 em
33.
2
_!_=_1 __ __!__ = 10 + 1 = 18 = 2_
v 0.8 -1 8 8 4
4
or v=-m
9
34. Length of image = (-f-)b
f-u
1 1 1
35'. -+-=-
u v f
_du_dv=O
u2 v2
dv du
or

or
dv v
2
du 10x10 _
1
_
1
-=---=----x9rns_ =-lms
dt u
2
dt 30x30
922 Chapter 23 Ray Optics
36. Total deviation
37.
38.
39.
40.
41.
But
or
= (180-2a) +
=
= 180

:. Total deviation = 360-28
O I
I-X x-1
O
I
I
1--x-v-t-x-v--.j
As is clear from figure, the new distance is 2x - 2v. The
distance of image from object is reduced by an amount 2v
in orie second.
first images is due to r,eflection from the front surface
ie unpolished surface of the mirror. So, only a small
fraction is the incident light energy is reflected. The
second image is due to reflection from polished surface.
So, a major portion of light is reflected. Thus, the second
image is the brightest.
10 em
1---1
10 em

I I' 0 0'

Observer
1--- x+y-10 ---1
As is clear from figure the distance of image with reference
to observer reduces by 10 em in one second.
Clearly, the distance of image from observer is 40 em.
0 ._1_0_e_m-+!i--em-ef
30 em
Observer
,
42. Clearly, i + r = i + i = 140
or i = 70
Clearly, plane mirror makes an angle of 20 with vertical
and 70 with horizontal.
43. !_ = l_
d u
or
44.
d
d
I = - f or I = 8f.
u
45. m=_j_
f-u
. If m = + 3, then
-24
3=--
-24-u
or -24- u = -8 or u + 24 = 8.
or u = 8 -24 em= -16 em
If m = -3, then
-3 =--=3.._
-24-U
u+24 = -8
or u = -32cm.
46. Only the light-gathering power is reduced.
47. Think in terms of rectangular hyperbola.
48. P=(!l-1)(2__2_)
Rl Rz
J..l decreases, p QJ.e.Jreases andf
49. Power of the system decreases due to separatidn between
the lenses. So, the focal length increases.
50. Note that two refractive indices are involved. Therefore,
two images will be formed.
51. Diminished erect image is produced only by a concave lens.
52. When convex lens is surrounded by denser medium, it
behaves like a diverging lens.
i
I
I
I
I
/
53.
A !2 fd2
1 1 d
.. fl+f2=fd2
1 1 d
20-56 = 20 (-56)
56-20 d
20x56 -20x56
d = -36 em
54. As seen from a rarer medium (L
2
or L
3
) the interface L
1
L
2
is concave and L
2
L
3
is convex. The divergence produced
by concave surface is much smaller than the convergence
due to the convex surface. Hence, the arrangement
corresponds to concave-convex lens.
55. ..!.. = (1.65 -1)(.3._)
f 40
56.
1 0.65
or -=--
or
n=-'-
f+u
! 20
20
!=-em
0.65
= 30.77 em "' 31 em
f+u=L
n
or u = f = C n )'
or u =-( )!.tul= f
57. m=-'-
f+u
58.
59.
1 -20 .
-=---or -20+u=-40
2 -20+u
or u=-40+20 or u=-20cm
I v
0 u
I -25
-=-
15
I= 15 x 2.5 em= 3'7!5 em
..!.. = _!_)
f rl r2
' r""'
For lens to be concave, (_!_ _
rl r2 . J.)
. : . 1 1
or ->- orr
1
<r
2
.rh ,.
rl r2
1 1 1
60.
' 2.. 1 1 1
, -=---
! -4 -10
Chapter 23 Ray Optics 923
1 1 1
or -=---
! 10 4
1 2-5 3
or -=--=--
! 20 20
20
or f = -- em= --6.67 em
3
The negative sign indicates that the lens is concave.
1
61.
f -
m=--=-2=-3-
f+u 1
- +U
3
2 1
---2u=-
3 3
1 2
or -2u = - +- = 1
3 3
1
or u = --m = -0.5m
2
1 1 1
62. ----=-
v -f/2 f
1 1 2 1
-=---=--
v f f f
or v = -f
v -f
Again, m=-=--=2
u -f/ 2
Clearly, the image is virtual and double the size.
63. ..!_ = or f = _R_
f R 2(1!. -1)
Now,f> R
R
--->R

1
or ---> 1 or 2(!1-1) < 1
2(!1-1)
or < % or 11 < ( 1 +%)
or 11 < 1.5.
64. Clearly, 2f = 20 em orf = 10 em
Now, u = -15 em, v = ?
F = 10 em
1 1 1
v -15 10
[ 1 1 1 1 1
or -+-=-or-=---
v 15 10 v 10 15
1 3-2 1
or -=--=- orv = 30 em
v 30 30
The change in image distance is (30- 20) em ie, 10 em.
65. The power of the given system is a combination of the
positive power of the convex lens, negative power of the
plano-concave lens of water and zero power of the plane
mirror. Clearly, the power of the system decreases.
66. 5 = (1.5 -1)( *) ... (i)
... (ii)
924 I Chapter 23 Ray Optics
Dividing Eq. (i) by Eq. (ii), we get
_
5
= 0.5n
1.5 -n
or -7.5+5n=0.5n or -7.5=-4.5n
75 5
or n=-=-
45 3
67. _
40 R
1
R
2
1 1 1
R
1
R
2
20

' f 2 20
or J =- 0;5 (- 210 J
1 1
or - =- or f = 80 em
f 80 \
It behave like a convex lens of focal length 80 em.
1 1 1
68. ---=-
v u f
or
or
or
69. m=-f-
f+u
Now,
or
Again,
or
1 1 1
v -15 10
1 1 1
-=--+-
v 15 10
1 -2+3
-=--
v 30
v = 30 em
+2=_1__-
f-10
2f- 20 = f or f = 20 em
20
-2=--- or-40-2u = 20
20+u
-2u = 20 + 40 or u = - 30 em
70. The condition for achromatism is
ffit P1 + WzPz = 0
ffit P1 = -wzPz
=>
W1 = _ Pz
Wz P1
Now, P
1
+ P
2
= 2D
or 5 + P
2
= 2 or P
2
= -3D
ffit -3 3
-=--=-
(1)2 5 5
_ !11 llz _ !13 _ _ 1
71. zll1 X3 llz X4 113 --x ----4 !11 --
llz 113 !14 1!14
72. The central ray goes undeviated. So, J..lz = J..t
1
.
Also, J..t
3
< J..tz
c VA A
73. n
1
=-=-=-
v1 vA
1
A
1
c VA A
nz "' -=-=-
v2 vA
2
A
2
76.
;1 = -
1
)(
D
. 'd' /
1
(n -1)n'
lVl mg, - = _:____________:___
f n-n'
or fl = -1)
n -n
77. llg sinec = 11
1
sin 90
or llg sin ec = 1
When water is poured,
llw sin r = lls sin ec or llw sin r = 1
Again, lla sine = llw sin r
or lla sine= 1
or sine = 1 or e = 90
sinr 1
78. -.-:- = tan30= r;::;
smz v3
sini r;::; r;::;
or - .-=v3 or !l=v3
smr
So, speed of light in Y is .J3 times less.
79.
1 1 1 1 2 2
or 1-- = -,--- or - = 1-- =- or 11 = -
11 3 ' 11 3 3 3
1 3
Now,--=-
sinic 2
. . 2 . . _1(2J
or sm zc =
3
or zc = sm
3
or ic = sin-
1
(0.67).
[
80. Wavelength in vacuum,
A =
3
X
108
X 10
10
A= 0.6 X 10
4
A
Sx10
14
= 60ooA
'A
Now,
or
6000
A=4oooA
M
=fo
. 81.
f e
11 1.5
9 = fo or f. = 9f
fe o 'Je
Also, L = fo + fe or 20 = fo + fe
or 20 = 9 fe +f. or 20 = 10 fe
or fe = 2 em
:. f
0
=9x2cm=18cm
. . 1 1 1
82. FortheobJectlve, vo- _
11 3
_
8
=
11 4
or _!_+ 3.8 = 4 or_!_= 0.2 = .!_
or
5
Now: M =--= -19
' 0 1
3.8
Again, M = M
0
X Me
95
-95 =-19 xMeor Me=-=5
19
83. L = v
0
+ fe -==- v
0
= L - f. '
or V
0
=19.2cm
1 1 1
19.2
uo
1.6
1 10 10
or
--::::::---
uo 16 192
1 120-10 100
or -
uo
192 192
192
or u =--cm=-1.75cm
0
110
84
. 'A2 = 4800 =
0
_
8
'A
1
6000
5
New resolution limit = 0.8 x 0.1 mm = 0.08 mm
a= 3.5x10
3
rad = 3.5 x 180
85. 3.8x10s 3.8x100 7t
86.
3.5x180x7
38x100x22
Also M = fo =
400
= 40
' f. 10
!2
tcx:-
d2
40x35x180x7 =
2
l.lo"'
21
o.
3Sx100x22 '
j_ means that the diameter of aperture is L.
d ' 2
Chapter 23 Ray Optics I 925
1 !2
Now: -ex:--
' 100 / f J
2
l2
1
or - cx:4
100
!2
Again, t ex: ( f J
or t ex: 64
Dividing Eq. (ii) by Eq. (i),
64 16
100t=-=16 or t=-s
4 100
87. Shift
1=3(1-.! ) or .!=1-.!_
Jl 3 11
1 1 2 3
or - = 1-- =- or 11 =- = 1.5
Jl 3 3 2
1 1 1
88. !=--;;--;;_
or
or
1 1 1 100 25
2.5=---- or-=----
-0.75 u u 75 10
1 4 5 1 -8-15 23
-=--- -or-=--=--
u 3 2 u 6 6
6
u = --m = -0.26m
23
; feD
89. Separat10n =
10
+-F--
J e +D
=
80
+ Sx25 =
80
+ 125
5+25 30
= 84.16 em= 84.2 em.
90. (1) M =-J
0
(1 + f. J
fe D
M=-
5 25
M = -40( J = -40x% = -48
(2) M = fo = - 200 = -40
fe 5
91. For reading purposes
u =- 25 em, v =-50 cm,J = ?
1 1 1 1 1 1
-=---= -+-=-
.. f v u 50 25 so
p = 100 = +2D
f
For distinct vision,
f' = distance of far point = -3m
P = _!_ = _ _!D =- 0.33 D.
f' 3
1 1
92. f=-=-m=20cm
p 5
1 1 1
Now, ---=-
v u 20
... (i)
... (ii)
I
926 I Chapter 23 Ray Optics
1 1 1 1 1 1
or ---=-or-=- - -
v -25 20 v 20 25
1 5-4 1 1
or
= 100 or = 100
or d = lOOcm =1m
93. For the eye-piece
ve = -25 cm,f, = 5 em
1 1 1
---=-
-25 ue 5
1 1 1 1 -1-5
or -=----or-=---
ue 25 5 ue 25
25
or u =--
e 6
Now, v = L-lu I= 20-
25
o e
6
120-25 95
---em=- em
6 6
1 1 1 1 6
Now, ----=- or -=--1
95 / 6 uo 1 uo 95
or
94. When final image is formed at oo ,
M-
Now, v;; = 16- fe = 16-2.S= 13.5 em
M=


-0.4 2.5 .
95. Power of combination P = P
1
+P
2
96.
97.
For image at infinity
= + 20- 10 = + 100
F= _!_=_!_m=10cm
p 10
M = !?_ =
25
= 2.5
F 10
M = fo ,10 = fo ,f
0
= 200cm.
f. 20
--
Tij 2

. A
sm-
2
. (A+om)
sm --
A 2
cot- = ---'-------'-
2 . A
sm-
2
A . (A+om 1
cos
2
= sm -
2
- J
. A ' . A
or
sm- sm-
2 2
or
. ( 1t A ) . (A + om )
sm
2
-
2
=sm -
2
-
or
Jt A A om
---=-+-
2 2 2 2
or
om
, 2 2
98.
7t-2A =om
2 2
. . om = 180- 2A

.A
sm-
2
or
or
sin
J2 = --'1..---...=-----.J.
1 . (60+om)
J2 =sm
2
. 45o . (60o+om)
sm =sm
2
om= 30
. A+om
l=--
2
= 60 + 30 = 90 = 450
2 2
COt 1
99. -=-
100.
2
Now, !1 =-COt = _.!
fz 2
or !
2
=-2!
1
1 1 1
Now, -=-+-
F !1 fz
1 1 1
-=-+--
so fl -2!1
or 50f= -2+1 =-1-
-2!1 2f1
or 2j
1
= SO or f
1
= 25 em
Again, f
2
= -2 x 25cm = -SOcm
following arguments lead us easily to the right choice.
(i) Angle between any two lines is the same as the angle
between their perpendiculars.
.. i=3Q
0
(ii)
1 sin30
1.5 sinr
. l.S 0 75
or smr=-= .
2
-'
or r =48.6
(iii) 9 = r- i = 18,6
Required angle = 2 x 18.6 = 37.2
101. A = r
1
+ r
2
102.
103.
30 = r
1
+ 0
or r
1
= 30
Now: sin i = J2
' sin30
. . {;:;2 1
or sml = v.t.X-
2
. . 1
or
Slnl= J2
or i = 45
From ,:\,.abc, A+ 90 + (90- i) = 180
or i =A
Now, complementary angle at point d, 8 = 2 i
e = 2A
Only option (b) satisfies this.
-1 20+ 0.8 x 1o-14
ll
1
- (400 x 10-
9
)
2
or
or
or
or
or
-1 20+ 0.8x1o-14
Ill- 400x400x10
18
0.8
Ill= 1.20+-
16
Ill = 1.20 + 0.05
' . ' Ill = 1.25
. .
1
0 8
Slnl =--= .
c 1.25
2.
,
or
Again,
or
or
or
or
Chapter 23 Ray Optics 927
ll2 = 1.232
sini =-
1
-=0.81
c 1.232
ic = sin-
1
0.81
= 54.26
Now, sin 8 = 0.8 or 8 = 53.13
This angle is clearly greater than critical angle
corresponding to wavelength 400 run. So, light of 400 nm
wavelength undergoes total internal reflection.
104. For passing the ray from prism,
. . A
)l < cosec
2
(
90)
)l <cosec Z
ll < J2
llmax = J2
105. When an object is placed between 2f andf (focal length)
of the diverging lens, the image is virtual, erect and
diminished as shown in the graph. To calculate the
distance of the image from the lens, we apply
1 1 1 1 1 1
-=---=>--=---
! v u -20 v 30
V=
(20) (30)
20+30
= - 12 em (to the left of the diverging lens.)
121. Chapter 23 Ray Optics
Exercise II
1. As there is no deflection between medium 1 and 2.
Therefore, J.1.
1
= J.1.
2
2. Imaae can be formed on the screen if it is real. Real image
of reduced size can be formed by a concave mirror or a
convex lens.
Let u = 2/ + x, then
1 1 1
-+-=-
u v f
1 1 1
=> --+-=-
2/+x v f
1 1 1 f+x
=> -=----=
v f 2f +x j(2f +x)
j(2f + x)
=> v= .
f+x
It is aiven that u + v = 1.0 m
2f +x+ j(Zf +x) = (2f +x)[1+-
1
-] < l.Om
f+x ,f+x
(2j +X)2 1 0
or < . m
f+x
or (2f +x)
2
< (f +x)
This will be true only whenf < 0.25 m.
3. Given, A = 60
i = i' =I' A = 45
4
i+i'=A+o
or 90= 60 +8
() = 30
A
N()te that i = i' is the condition for minimum deviation.
Hence, 8 = 30 = <>min .
4. Image will be real.
We know that
1 1 1
-=---
! v u
2'_=1-2'_
f u
2'_=1+m
f.
v= j(m+1)
. s.
f 1!2 R1 R2
( :. u is negative)
or
1 1 2
or orf= 1.20m
f 8 0.3
6.

=4x3=12 em.
111 x 11x
7. -=-+---- =---+----z
f !1 !2 fd2 !1 !1 !1
1 X
=> f > 0 for every x.
-=-
f !12
For X=O,J=oo __..
Hence, for x = 0, system will behave like a glass plate.
8. We know that
or
11= and i+r=90
smr
r = 90- i
sini .
11 = = tant
sin(90-i)
or i =

= tan-
1
(1.62).
1 2 2 1
9. Used= f; where,-= = (1.5-1)x- =-
f R 20 20
=> f= 20 em
10. Using equation, the total apparent shift is
or s=4(1--
1
)+6(1--
1
)
4/3 3/2
= 3.0 em
Thus, h = h
1
+ h
2
- s = 4 + 6- 3
= 7.0 em.
11. We know that
velocity of light in vacuum
11
= velocity of light in water
4 3x10
10
3 velocity of light in water
=>velocity of light in water = 2.25 x 10
10
cms-
1
Time taken =
500
X
100
= 2.22 X 1 o-
6
s
2.25x10
10
Equivalent optical path = Jl x distance travelled in water
= ix500 = 666.64m
3
I
i
I
I
12. Only one converging point is found by this lens. Therefore,
only one image is formed.
13. According to Cartesian sign convention
u = - 40 em, R = - 20 em
l-11 = 1, 11
2
= 1.33
Applying equation for refraction t hrough spherical
surface, we get
f..lz _ f..l1 = f..lz - f..l1
v u R
1.33 1 1.33
-----=--
v -40 -20
After solving, v = - 32 em.
The magnification is m = hz = f..lJ v
h1 f..l zU
h
2
= _ __ 1(32)
1 1.33(-40)
or h
2
= 0.6cm
The positive sign shows that the image is erect.
14. This is a modified displacement method problem.
a+d 2
Here, a = 1.8m and --=-
a -d 1
Solving we get d = 0.6 m
a2 -dz

=0.4-m.
15. (i) When angle of prism is 'Small and angle of incidence is
also small, the deviation is given by 8 = (f.1-1)A. Net
deviation by the two prisms is zero. So,
(ii)
or
Flint
Crown
8
1
+8
2
=0


-1)A
1
+ (f.1
2
-1)A
2
= 0.
Here, 11
1
and 11
2
are the refractive indices for crown
and flint glasses respectively.
H
1.
51
+1.
4
g 1.5 and
ence, f..l
1
=
2
- 1. 77 + 1. 73 -1 75
f..l2- 2 - .
This givesA
2
=- 4
Hence, angle of t1int glass prism is 4. Negative sign
shows that flint glass prism is inverted with respect to
the crown glass prism.
Net dispersion due to the two prisms is
= - f..l r
1
)Al + (f..lbz - f..l r
2
)A2
= (1.51-1.4g) (6) + (1. 77 -1.73) ( -4)
= -0.04.
:. Net dispersion is- 0.04
t
Chapter 23 Ray Optics 929
16. As shown in Fig. (a)
T3: 2 TO
0
(a) (b)
In this case refraction of the rays starting from t
0
takes
d
place from a plane surface. So, we can use dapp = actual
or 3=
f..l
4
or J.1=:3
f..l
As shown in Fig. (b). In this case refraction takes place
from a spherical surface. Hence, applying
f..l2 _ f..l1 = f..lz - f..l1
v u R
we have,
1 4/3 1-4/ 3
--=---
(-25 / 8) -4 -R
1 1 8 1
or -=---=-
3R 3 25 75
. . R = 25 em.
Now, to find the focal length we will use the lens maker
formula
17. Here, x = u + v
f f -v
m=--=--
f+u f
and image is real, magnification is negative.
f -(m+1)f
-m=-- u=
f+u' m
f-v
From
f
Put in Eq.(i)
- (m +
1
) f ( 1)f
x= + m+
m
Solving, we get, f = mx
2
(m+1)
1 1 sini
18. sin C=-=----
f..l sinr' I sini sinr'
As is clear as shown in figure
LCBD=goo
goo -r +goo -r' = goo
or
sini
:. sin C = _ ___:____
sini
sin(g0-r) cosr
sini . ( . )
=--. =tant : t=r
COSt
C = sin-
1
(tani) = sin-\tanr).
i r

B
Rarer
930 Chapter 23 Ray Optics
19. Let r be the radius of circle through which other objects
become visible. The rays of light must be incident at
critical angle C.
. C 1 r
sm
J..lzrz = rz + hz
(J..lz -1)rz = hz
Diameter
h
r=---

2h
2r=
20. Here, i
1
= 60 ,A= 30,8 = 30
As i
1
+ i
2
-= A+ 8,
i
2
= 0
Hence, angle between the ray and the face from which it
emerges = 90-0 = 90.
21. As a convex lens alone can form a real images as well as a
virtual image, therefore, the lens in the present question
is a convex lens. Let f be the focal length of the lens and
m be the magnification produced.
In the first case, when image is real,
As
u = -16cm, v = (m x 16)cm
1 1 1
---=-
v u f
1 1 1 1 16
--+-=-or 1+-=-
16m 16 f m f
In the second case, when image is virtual,
u = -6cm, v = ( -6m)cm
1 1 1
From---=-
v u f
1 1 1 1 6
--+-=-or 1--=-
-6m 6 f m f
Add Eq. (i) and Eq. (ii) we have
22 22
2=- or f=-=11 em.
f 2
.. (i)
.. (ii)
22. When a slab of thickness tis introduced between P and
the mirror, the apparent position of P shifts towards the
mirror by ( t- Hence, the mirror must be moved in
the same direction through the same distance.
23. In the situation given, the image will be formed at infinity,
if the object is at focus of the lens ie, at 20 em from the
lens. Hence, shift in position of object
X = 25- 20 = ( 1 - J t
5 = (1 - J:__) t
1.5
t = 15 em.
,
24. The combination of two lenses is
As
1 1 1
- =-+-
F f1 fz
' -
_!_ = (J..l
1
-1)(2_ + _!_J+ (J..l
2
-1)(J:__- _!_J
F oo R -R oo
- Jll -1 J..lz -1
---+--
R R
_!_ = Jl1- J..lz
F R
or F=-R-.
Jl1- J..lz
25. As is clear from figure , A = 30, i
1
= 60
As the ray retraces its path on reflec-
tion at the silvered face, therefore,
i
2
= 0, r
2
= 0
As r
1
+r
2
=A
r
1
+ 0 = 30
or r
1
= 30
J..l= sini1 = sin60 = .J3;2 =.J3.
sinr
1
sin30 1/2
26. For the relaxed eye, magnifying power is

" uo fe
-45 =-VoX 25' Vo = 9.
uo 5 uo
For objective lens, image is real.
v
Vo=+vo,Uo= - ;.
Given, f a = 1 em.
1 1 1
From-- -=-
vo Uo fa
1 9 1
-+-=-;v
0
=10cm.
vo vo 1
Length of the tube = V
0
+ f, = 10 + 5 = 15 em.
2 7. For total internal reflection at AC face
sini:::: J.lw
J.lg
sine::::-
4
-
3xl.5
. e s
sm ::::-
9
28. Note that image formation by a mirror does not depend
on the medium. As P is at a height h above the mirror,
image of P will be at a depth h below the mirror.
If d is depth of liquid in the tank, apparent depth of P,
d-h
Xl=--
J..l
:_ Apparent distance between P and its image
d+h d-h 2h
=Xz-Xl =-----=-
J..l J..l J..l
I
x lOx
c=-v=--
tl ' tz
29.
sin C' = _.!_ = =
10
x x L
11 c t
2
x
e
, . _
1
(10t1 J
=Sill --
tz
.. . (i)
30. As _.!_ = (!l- 1) (__.!__- __.!__)
f R1 Rz
:. __.!__=(1.5-l)(I__I_)
20 = R
1 -1
-=- R=-10cm
20 2R'
Refraction from rarer to denser medium
_&+llz =llz-111, where u==,v=f
u v R
1.
5
-l =__.!__ f=30cm.
f 10 20'
31. Final image is formed at infinity if the combined focal
length of the two lenses (in contact) becomes 30 em
1 1 1
or -=-+-
30 20 f
ie, when another concave lens of focal length 60 em is
kept in contact with the first lens. Similarly, let J..l be the
refractive index of a liquid in which focal length of the
given lens becomes 30 em. Then
210 =(%-1 ... (i)
310 = ( 3 2 - 1 )( - ... (ii)
From Eqs. (i) and (ii), we get
9
11--
- s
32. For convex mirror (positive focal length) image is always
smaller in size. For concave mirror (negative focal length)
image is smaller when object lies beyond 2f.
33. Total internal reflection takes place when ray of light
travels from denser to rarer medium.
F h

8
llz d .
8
113
urt er, Sill
12
= - an Sill
13
= - .
Ill Ill
Since,
113
>
113
Ill Ill
812 > 813
Smaller the value of critical angle more the chance of
total internal reflection.
34. Using, llz -
111
= llz -Ill , we get
v u R
or
1.5 1.0 1.5 -1.0
v 20
v=60cm
' 1 1 . 1 -
35. As
sine sin45 11 -v2
:. Possible value of J..l are 1.5 and 1.6.
,
Chapter 23 Ray Optics 931
36. In air
1 . ( 1 1 )
p = f = (1.5 -1) Rl - Rz = 2
_!_J= _3_ = 4
R
2
0.5
In water
P
' ( 1.5 1) 4 4.5-4
=
4 1 3
- X =-
4
-x4=0.5D.
A+B 1
37. =>
Therefore refrective index for A. 4000A will greater
then 1.5.
38. Use _.!_ = - 1 )[__.!__ - __.!__]
f !lm Rl Rz
Since (IlL< !lm) because lens is of water and IlL= llw
39. !l=-1-= 1 _1 __ _
sine sin48.6 0.75 3
40. Light cannot undergoes total internal reflection when it
is travelling from air to water, ie, from rarer to denser
medium.
41. sine=_.!_= -
1
- = 3. = 0.6667
ll 3/2 3
e = sin-
1
(0.6667) = 41.8
42 F
1 el
. rom ll = -.-, s1n =-
Sill e !l
As llv > !lr
ev < e,.
43. As shown in figure
OS= h
When angle of incidence
is slightly greater than
e, light undergoes total
internal reflection.
:. Diameter of circle of
light coming from water
surfaee = 2 r = 2 (OB)
= 2 OS tan e = 2h tan C.
----- -----
----- -----
------ -----
h c
-------- --------
-------- --------
--- ------ --------
--------- ---------


44. In air or water, a convex lens made of glass behaves
as a convergent lens but when it is placed in carbon
disulfide, it behaves as a divergent lens. Therefore, when
a convergent lens is placed inside -a transparent medium
of refractive index greater than that of material of lens, it
behaves as a divergent lens.
It simply concludes that property of a lens whether
the ray is diverging or converging depends on the
surrounding medium.
45. If a mirror is placed in a medium other than air its focal
length does not change asf = R/2, but for ,the lens
_.!._ = Cang -1)(__.!__ _ __.!__)
1
fa Rl Rz
and __.!__ = C.vng -1)(__.!__ _ __.!__)
fw Rl Rz
As w ng <
0
ng, hence focal length oflens in water increase.
The refractive index of water is 4/3 and that of air is 1.
Hence, llw > lla
932 Chapter 23 Ray Optics
46. The magnifying power of telescope in relaxed state is
m
= fo . ,
) '
h .'.s
So, for high magnification, the focal length of objective
length should be larger than of eye-piece.
Resolving power of a telescope = _d_: ..
1.22A
For high resolving power, diameter (d) of objective should
be higher.
47. We know that power of lens is a reciprocal of its focal
" 1 1 .
length, hence P = - =
50
= 2 D
f-
100
Since, lens is concave hence, its power will be 2D.
If the objective is placed at infinity then
u==,v=?,f=50cm
1 1 1
From the formula, --- = -
v u f
1 1 1
v 00 -50
v=-SOcm
Thus, concave lens will form an image of the object at
infinity at a distance of 50 em.
48. When glass surface is made rough, then light incident
on it is scattered in different directions. Due to which its
transparency decreases.
There is no effect of roughness on absorption of light.
49. Refraction index of diamond w.r.t. liquid zlld = -_-
1
-
, smC
or
.J6 1
.J3 =sine
" 1 " so
smC=-=sm4
F2
c = 45
50 Red glass transmits only red light and absorbs all the colours
of white light. Thus, when green flower is seen through red
glass it absorbs the green colour, so it appears to be dark.
51. Here, u = -mf, u = v,f = -f
52.
Using lens formula !_..!. = ..!., we have
v u f
1 1 ( 1) -1 1
-=-- 1+- =-(m+1)=-(m+1)
v f m mf u
or
v 1
or -=--
u m+1
. 'fi . v 1
Lmear magm catiOn =- = ---
u (m+ 1)
The lens formula is
1 1 1
-=---
! v u
... (i)
The graph between u and vwill be curve as shown in figure.
Let the coordinates of P be (x, x), then u = -x and v = x
1 1 1 2
FromEq.(i), -=---=- orx = 2f
f X -X X
:. Coordinate of point Pare (2f, 2f) .
f
53. When lens is in air, then
..!_=(aJ.l. -1)(_!_ _J_J.
f g ' R R
' l 2
1 ' (1 1J ---
10 R
1
R
2
.
or
or
( ;1-
When lens is in water, then
= r:Jlg -1J(_!_ _ _!_J
f llw Rl Rz

1.33 5 40
f'=40cm
54. For a telescope
I= fo
a fe
= _Q2_ => = so
0.5 0.03
55. !tis observed if
4
and
Here,
and
. _A+om
[=---.
2
LA= 60
L
. 3
t=Le=-LA
4
o = 30
m 4
56. Incident ray and finally reflected ray are parallel to each
other means, o = 180
From
=>
=>
1
57. Jl oc-
A.
0=360-20
180 = 360 -20
e = 90
1-lrarer < 1-ldenser
Actenser < Ararer
ie, wavelength decreases.
58. allg =-.-
1
-=>sinC=-
1
-.
smC allg
As J..l for violet colour is maximum, so sin C is minimum
and hence critical angle C is minimum for violet colour.
59. As A.r > lvv :. ov > or
The statement given is true neither, in prism spectrum nor
in grating spectrum.
I
I
I
60.
61.
_!_ = (__!_ - __!_ J
F R
1
R
2
-
1
- = (1.5 -l)(_!_+_!_J
0.06 R
1
R
2
(Taking R
1
positive and R
2
negative)
1 1 1 100
-+-=----
R1 R2 0.06 X 0.5 3
R
1
1
Now, -=- or R
2
= 2R
1
R
2
2
3 100
2R
1
3
9
R
1
=
200
= 0.045m
R
2
= 2R
1
= 2 x 0.045 m = 0.09 m.
sinC = _!_ = -
1
- = = 0.75
11 4 / 3 4
c = 48
:. Angle with horizontal = 90- C
= 90 - 48 = 42
. .
62. AI> it is clear from figure distance travelled in. glass place
=DB= OC
cosr
OB=-t_ .
cosr
90
63. AI> the ray turns through 90, therefore, C =-= 45
2
lin =J2=1.414.
sin C sin 45 1/ -v 2
64. p = _!_ = (!l-1)(_!_ _ _!_J
F R
1
R
2
66.
= (1.4 -1)(_!_ + 100) = 40 = 2.66D
00 15 15
sin(A+om)/2 sin(60+60)/2
=
sinA/2 sin60 /2
= sin60 = .J3 = 1.
73
sin30 .2xl / 2
6 7. From the lens formula !.. -!.. = _!_ = constant
v u f
u is always negative, vis positive.
68. The distance in the experiment are measured by a vernier
scare provided on the microscope.
-j t i
Chapter 23 Ray Optics 933
' '
R1 . Rz
."',. ! I I

20 -20
1 1 1
=-+-=-
40 120 30
4
f = -x30 = 40cm
3
70. The angle subtended at the eye becomes 10 times larger.
This happens only when the tree appears 10 times nearer.
l .
71. 8=-,8=1 mm.
r
So,
and
1 1t 1
8=-=-x- rad
60 180 60
1=3m
l
x=r=-
8
3
1t 1 = 10 krn.
- x-
180 60
72. For dispersion without deviation
8
1
- 8
2
= 0, ie , 8
1
= o
2
Cl-11 -l)Al = -l)A2
or (1.54 -1)4 = (1.72-1)A
2
73. The image formed by a concave mirror is always real
when the object is placed between focal point and infinity.
But when the object is placed between focal point and
pole, then the image is always virtual.
But when the object is virtual ie, object is placed far away
from the focal point, then the image is certainly real.
Hence, the required answer is certainly real if the object
is virtual.
74. Given that,
the refractive index of the lens w.r.t. air, aflg = 1.60 and
the refractive index of water w.r.t . air aflg = 1.33
The focal length of the lens in air, f = 20 em.
We know that for a lens
_!_ = (__!_- __!_ J
f R1 R2
When the lens is in the air
__!_ = ( -1)(_!_- __!_ 1
20 a g R
1
R
2
)
or -=(1.60-1) --- 1 ( 1 1 J
20 R
1
R
2
or __!_ = 0.6ox(_!_ _ _!_J
20 R
1
__ R
2
When the lens is in the
... (i)
__!_=( 11 -l)(_!_ _ _!_J
f' w g R1 R2
I
934 Chapter 23 Ray Optics
or


or -\ = ( a!-lg -a 1-lw )(J__J_)
f a!-lw R1 Rz
]__ = (1.60 -1.33 )(__!_ _ __!_)
f' 1.33 R
1
R
2
or ]__-
27
(__!_ _ __!_ ) ... (ii)
f' 133 R
1
R
2
On dividing Eq. (i),by Eq. (ii), we get
.[_ 0.60x133
20 . 27
or f' = 20x2.95cm 60cm.
Hence, its focal length is three times longer than in air.
d 25
75. M=1+-=1+-=6
f 5
76. The limit of resolution of an optical instrument arises on
, account of diffraction of light.
77. om= A,!J- = 1.5,A =?
sin(A + om)12 sin(A + A)l2
1-1 = sinAI2 = sinA12
2 sin A I 2 cos A I 2
sinAI2
3 A
-= 2cos -
2 2
A 3
cos-=-
2 4
= cos-
1
(%}A= 2cos-
1
(%)
78. In displacement method,
size of object, 0 =

1
2
. . 3
2
=9xi
2
cm
J
2
= 1
79. Shift


1
=6x -=2 em
3
80. Length of tube 10 em
f a + fe = 10 em
Magnification m = f o = 4
f e
f o = 4fe
Putting in Eq. (i),
I
5fe-10 em
or fe = 2 em
and f a= 8 em
fa = 8 em, fe = 2 em
Hence, L
4
and L
1
will be used.
,
81. Bi-convex lens is cut perpendicularly to the principal axis,
it will become a plano-convex lens.
Focal length of bi-convex lens
]_ = (n -1)(2_- __!__)
f R1 Rz
1 2
-=(n-1)-
f R
f=-R-.
2(n -1)
... (i)
For plano-convex lens
]__ = (n -1)(]_- .!..)
f1 R =
R
!
1
= (n-1)
.. . (ii)
Comparing Eqs. (i) and (ii), we see that focal length
becomes double.
1
As power of lens P ex: -,--......,-,---:-
focal length
Hence, power will become half.
New power = . = 2D
2
82. In normal adjustment,
L = fo + fe = 200 + 4 = 204 em.
83. For o = (!J-- 1)A
As 1-lv = 1.525 and 1-lr = 1.520
ie
1-lv > 1-lr
ov >or
or D
2
> D
1
or D
1
> D
2
84. As object is at the centre of the sphere, the image must be
.
:. Distance of virtual image from centre of sphere = 6 em.
. 1 1 8 6
85. From smC=-=--=0.75,C=4.
0
1-1 1.33
86.
d = 2 x tan C = 2 x 8 tan 48.6
= 16 x 1.13 = 18.08 em
1inm -4
A=--=5x10 mm
2000
= 5 x 10-
7
m = 5000 A
A.'= 2:. =
5000
= 40ooA
1-1 1.25
87. When a ray of light moves from one medium to other,
its velocity changes. This change depends on refractive
index of the medium. Light travels from denser to rater
medium, ie, from medium of higher refractive index to
lower refractive index. So, in second (rarer) medium, its
velocity in.creases.

.'

You might also like